UPSC IAS Prelims 2024 Answer Key: GS Paper – 1 | SET A,B,C and D [Detailed Explanation]

UPSC Prelims 2024 Answer Key prepared by Rau’s IAS faculty, contains the detailed analysis of the IAS Prelims Exam 2024. By matching your attempted answers with this answer key, you can estimate your score in the Exam.

UPSC Prelims 2024 Answer Key & Explanation for GS Paper 1

Qn. No. 1
SubjectIndian Polity
QuestionHow many Delimitation Commissions have been constituted by the Government of India till December 2023?
(a)  One
(b)  Two
(c)  Three
(d)  Four
Answer(d)
 ExplanationIn India, Delimitation Commissions have been constituted 4 times: 1952 under the Delimitation Commission Act, 19521963 under Delimitation Commission Act, 1962,1973 under Delimitation Act, 1972 and2002 under Delimitation Act, 2002.
More Information
Reason/ Motivation of question Delimitation exercise was in news in the context of Assembly Elections to be conducted in Jammu and Kashmir.
Also, the implementation of Women’s reservation (Nari Shakti Vandan Adhiniyam) is to be done post the delimitation.
The delimitation of constituencies for the Lok Sabha and State Legislative Assemblies is to be carried out on the basis of the first Census after 2026.
The 2021 Census was originally postponed due to the COVID-19 pandemic and subsequently due to delays on the part of the Central government
Primary Source of Info.https://www.eci.gov.in/Documents/Delimitation/FAQ-Delimitation.pdf  
Nature of questionFactual
 Difficulty levelEasy/Moderate
Qn. No. 2
SubjectIndian Polity
QuestionThe Constitution (71st Amendment) Act, 1992 amends the Eighth Schedule to the Constitution to include which of the following languages?
1.    Konkani
2.    Manipuri
3.    Nepali
4.    Maithili
Select the correct answer using the code given below:

(a)   1, 2 and 3
(b)   1, 2 and 4
(c)   1, 3 and 4
(d)   2, 3 and 4
Answer(a)
Explanation The Eighth Schedule to the Constitution consists of the following 22 languages:-
(1) Assamese, (2) Bengali, (3) Gujarati, (4) Hindi, (5) Kannada, (6) Kashmiri, (7) Konkani, (8) Malayalam, (9) Manipuri, (10) Marathi, (11) Nepali, (12) Oriya, (13) Punjabi, (14) Sanskrit, (15) Sindhi, (16) Tamil, (17) Telugu, (18) Urdu (19) Bodo, (20) Santhali, (21) Maithili and (22) Dogri.

The Constitution 71st Amendment Act  added three more languages; Konkani, Manipuri and Nepali in 1992.Bodo, Dogri, Maithili and Santhali were added by the Constitution 92nd Amendment Act in 2004.
 More Information
Reason/ Motivation of questionThere is an ongoing demand to include Khasi Language, Tulu, & Sign language in the eighth schedule of Indian Constitution.A Bench headed by CJI D.Y. Chandrachud has said recently, that the Court can’t direct Centre to recognise a language as ‘official language’
Primary Source of Info.https://www.mha.gov.in/sites/default/files/EighthSchedule_19052017.pdf

https://www.india.gov.in/my-government/constitution-india/amendments/constitution-india-seventy-first-amendment-act-1992
Nature of questionFactual
Difficulty levelDifficult
Qn. No. 3
SubjectHistory & Culture
QuestionConsider the following pairs:
S.N. Party Its Leader
1. Bharatiya Jana Sangh Dr. Shyama Prasad Mukherjee
2. Socialist Party C. Rajagopalachari
3. Congress for Democracy Jagjivan Ram
4. Swatantra Party Acharya Narendra Dev

How many of the above are correctly matched?
(a) Only one
(b) Only two
(c) Only three
(d)   All four
Answer(b)
ExplanationPair 1 is correctly matched: The Bharatiya Jana Sangh was an Indian nationalist political party. This party was established in 1951 in Delhi. Its three founding members were Shyama Prasad Mukherjee, Balraj Madhok and Deendayal Upadhyaya. 

Pair 2 is not correctly matched: Socialist Party has been the name of several political parties in India, all of which have their roots in the Congress Socialist Party during the freedom struggle. The original Socialist Party had its roots in the Congress Socialist Party (CSP. The Socialist Party was founded not long after India’s independence when Jayprakash Narayan, Rambriksh Benipuri, Basawon Singh (Sinha), Acharya Narendra Dev led the CSP out of Congress. 

Pair 3 is correctly matched: The Congress for Democracy (CFD) was an Indian political party founded in 1977 by Jagjivan Ram. It was formed after Jagjivan Ram, Hemvati Nandan Bahuguna, and Nandini Satpathy left the Indian National Congress of Indira Gandhi and denounced her rule during the Indian Emergency.

Pair 4 is not correctly matched: The Swatantra Party was an Indian liberal political party that operated from 1959 to 1974. It was established by C. Rajagopalachari in response to his perception that the Indian National Congress, under Jawaharlal Nehru’s leadership, was adopting an increasingly socialist and statist approach.
More Information
Reason/ Motivation of questionDue to the recent elections, these parties have been in news continuously. UPSC focuses on the topics going in the current affairs which are also relevant from the static point of view. These topics can be related to the Post-Independence portion of the UPSC syllabus.
 Primary Source of Info. 1.   India Since Independence-Bipan Chandra
2.Pre-Pareekshan-2 (2024) conducted by Rau’ IAS.
Nature of questionStatic
 Difficulty levelModerate
Qn. No.4
SubjectIndian Polity
QuestionWhich of the following statements are correct about the Constitution of India?

1. Powers of the Municipalities are given in Part IX A of the Constitution.
2. Emergency provisions are given Part XVIII of the Constitution.
3. Provisions related to the amendment of the Constitution are given in Part XX the Constitution.

Select the answer using the code given below:

(a)1 and 2 only
(b)2 and 3 only
(c)1 and 3 only
(d)1, 2 and 3
Answer(d)
 ExplanationFollowing are the respective parts of the Indian Constitution: PART IXA THE MUNICIPALITIESPART XVIII EMERGENCY PROVISIONSPART XX AMENDMENT OF THE CONSTITUTION
 More Information
Reason/ Motivation of questionTesting the basic understanding of the Constitution bare act.
Primary Source of Info.Constitution of India
Nature of questionFactual
 Difficulty levelModerate
Qn. No. 5
SubjectIndian Polity
 QuestionWhich one of the following statements is correct as per the Constitution of India?

(a)   Inter-State trade and commerce is a State subject under the State list.
(b)   Inter-State migration is a State subject under the State List.
(c)   Inter-State quarantine is a Union subject under the Union List?
(d)   Corporation tax is a State subject under the State List.
Answer(c)
ExplanationStatement (a) is incorrect as Inter-State trade and commerce is an entry number 42 in the union list. Statement (b) is incorrect as  Inter-State migration is entry number 81 of the Union list.
Statement (c) is correct as Entry 81 of the Union list states “inter-State quarantine”.
Statement (d) is incorrect as Corporation Tax is the entry number 85 of the Union list.
More Information
Reason/ Motivation of questionTesting the basic understanding of the constitution bare act. (Federal provisions)Linked with quarantine measures that were done during COVID crisis and subsequently was in news during the Farmers protest and Shaheen Bagh protest.
Primary Source of Info.Constitution of India
Nature of questionFactual
Difficulty levelDifficult
Qn. No.6
SubjectIndian Polity
QuestionUnder which of the following Articles of the Constitution of India, has the Supreme Court of India placed the Right to Privacy?

(a) Article 15
(b) Article 16
(c) Article 19
(d) Article 21
Answer(d)
Explanation The Supreme Court in a landmark judgment on 24 August, 2017 (K.S. Puttaswamy  v. Union of India) ruled that privacy is a fundamental right, and that the right to privacy is protected as an intrinsic part of the right to life and personal liberty under Article 21 as a part of the freedoms guaranteed by Part III of the Constitution.
 More Information
Reason/ Motivation of question  Recently, Right to privacy was in news in the context of Electoral bond judgement, Supriya Case (LGBT judgment). UPSC has continued with the trend of asking a question linked with expansion of a Fundamental Right by apex court in a landmark judgment.
Primary Source of Info.https://loksabhadocs.nic.in/Refinput/New_Reference_Notes/English/Right%20to%20Privacy%20as%20a%20fundamental%20Right.pdf
Nature of questionFactual
Difficulty levelEasy
Qn. No.7
SubjectInternational Relations
QuestionWhat are the duties of the Chief of Defence Staff (CDS) as Head of the Department of Military Affairs?

1. Permanent Chairman of Chiefs of Staff Committee
2. Exercise military command over the three Service Chiefs
3. Principal Military Advisor to Defence Minister on all tri-service matters

Select the correct answer using the code given below:
(a)1, 2 and 3
(b)1 and 2 only
(c)2 and 3 only
(d)1 and 3 only
Answer(d)
 ExplanationStatement 2 is wrong (CDS will not exercise any military command, including over the three Service Chiefs, so as to be able to provide impartial advice to the political leadership.)
More Information
Reason/ Motivation of question  Appointment of Anil Chauhan as CDS
Primary Source of Info.https://pib.gov.in/Pressreleaseshare.aspx?PRID=1597425

https://compass.rauias.com/current-affairs/chief-defence-staff-highlights-focusing-space-security/ 
Nature of questionCurrent
Difficulty levelModerate
Qn. No. 8
SubjectInternational Relations
QuestionOperations undertaken by the Army towards upliftment of the local population in remote areas to include addressing of their basic needs is called:

(a) Operation Sankalp
(b) Operation Maitri
(c) Operation Sadbhavana
(d) Operation Madad
Answer(c)
 ExplanationAs part of ‘Operation SADBHAVANA’, Indian Army is undertaking multiple welfare activities such as running of Army Goodwill Schools, Infrastructure Development Projects and Education Tours etc. for the children living in remote areas of Union Territory (UT) of Ladakh.

To improve the standard of education and to provide them with quality education, Indian Army is presently running seven (07) Army Goodwill Schools (AGS) under ‘Operation SADBHAVANA’ in Ladakh Region. More than 2,200 Students are currently studying in these schools.  

The funds are being utilised for various activities like Human Resource/Skill Development, Sports, Healthcare, National Integration, Infrastructure Development, Ecology, Environment and Education.
More Information
Reason/ Motivation of questionClimate Protest in Ladakh
Primary Source of Info.https://pib.gov.in/PressReleasePage.aspx?PRID=1895974
Nature of questionCurrent affairs
Difficulty levelEasy
Qn. No.9
SubjectGeography
QuestionThe longest border between any two countries in the world is between:

(a) Canada and the United States of America
(b) Chile and Argentina
(c) China and India
(d) Kazakhstan and Russian Federation
Answer(a)
ExplanationMap based. Factual. The international land border between the United States and Canada is the longest in the world at almost 8,900 kilometers. It includes the border between Canada and the continental U.S. as well as the border between Alaska and northern Canada.
More Information
Reason/ Motivation of questionIndo – Canada Issue in news
Primary Source of Info.Classroom program discussions
Nature of questionCurrent affair
Difficulty levelEasy

Qn. No.10
SubjectIndian Polity
QuestionWhich of the following statements about the Ethics Committee in the Lok Sabha are correct?

1. Initially it was an ad-hoc Committee
2. Only a Member of the Lok Sabha can make a complaint relating to unethical conduct of a member of the Lok Sabha.
3. This Committee (cannot) take up any matter which is sub-judice.

Select the answer using the code given below:
(a) 1 and 2 only
(b) 2 and 3 only
(c) 1 and 3 only
(d) 1, 2 and 3
Answer(c)
ExplanationStatement 1 is correct: During the sixteenth Lok Sabha, the Ethics Committee was constituted. Prior to this, it was a ad-hoc committee
Statement 2 is incorrect: Any person or Member of Parliament can make  a complaint relating to unethical conduct of a member of parliament.
Statement 3 is correct: The Committee on Ethics shall not take up any matter which is sub-judice and the decision of the Committee as to whether such matter is or is not sub-judice shall for the purposes of these rules be treated as final.
 More Information
Reason/ Motivation of questionMahua Moitra case
Primary Source of Info.https://loksabhadocs.nic.in/LSSCOMMITTEE/Ethics/Introduction/Introduction%20Ethics%20Committee.pdf

https://indianexpress.com/article/explained/ls-ethics-committee-its-constitution-members-8999613/
Nature of questionApplied
Difficulty levelmedium
Qn. No. 11 
Subject History & Culture
QuestionWho was the Provisional President of the constituent Assembly before Dr. Rajendra Prasad took over?

(a)   C. Rajagopalachari
(b)   Dr. B. R. Ambedkar
(c)   T.T. Krishnamachari
(d)   Dr. Sachchidananda Sinha
Answer(d)
ExplanationThe Constituent Assembly of India held its first meeting on December 9, 1946, in the Constitutional Hall in New Delhi, the capital city.

Dr. Sachchidananda Sinha served as the initial provisional president of the Assembly.Just two days later, on December 11, 1946, Dr. Rajendra Prasad was elected as the permanent chairman, with Harendra Coomar Mookerjee appointed as the assistant chairman.
More Information
Reason/ Motivation of questionImportant theme of Modern Indian history 
Primary Source of Info.A brief history of Modern India-Spectrum
Nature of questionStatic
Difficulty levelEasy
Qn. No. 12
SubjectHistory & Culture
QuestionWith reference to the Government of India Act. 1935, consider the following statements:

1. It provided for the establishment of an All India Federation based on the union of the British Indian Provinces and Princely States.
2. Defence and Foreign Affairs were kept under the control of the federal legislature.

Which of the statements given above is/are correct?

(a)   1 only (b)   2 only (c)   Both 1 and 2 (d)  Neither 1 nor 2
Answer(a)
ExplanationThe Government of India Act 1935, enacted by the British Parliament and receiving royal assent in August 1935, introduced several significant provisions. It aimed to establish an all-Indian Federation comprising provinces and princely states as units.

The Act delineated powers between the center and the units using three lists: the federal list, the provincial list, and the concurrent list.
However, the sections of the Act designed to create the Federation of India never came into effect due to opposition from the rulers of the princely states.

In the federal government, there is an appearance of responsible government.
However, this is superficial, as the direct powers vested in the governor-general over defence and external affairs significantly restrict the scope of ministerial activity. Additionally, the level of representation granted to the rulers of the Indian States effectively eliminates any chance of initiating democratic control.
 More Information
Reason/ Motivation of questionImportant theme of Modern Indian history.
Primary Source of Info.History of Modern India-Bipan Chandra (Old NCERT) A brief history of Modern India-Spectrum
Nature of questionStatic
Difficulty levelEasy
Qn. No. 13
SubjectHistory & Culture
QuestionWhich one of the following is a work attributed to playwright Bhasa?

(a)   Kavyalankara
(b)   Natyashastra
(c)   Madhyama-vyayoga
(d)  Mahabhashya
Answer(c)
ExplanationBhasa was a playwight and author of Madhyamavyayoga . Madhyamavyayoga falls under a particular type of Sanskrit drama called Vyayoga.
 More Information
Reason/ Motivation of questionImportant authors and their contributions
Primary Source of Info.Upindra Singh-History of Ancient and Early Medieval India
Nature of questionStatic
Difficulty levelDifficult
Qn. No. 14
SubjectHistory & Culture
QuestionSanghabhuti, an Indian Bhuddhist monk, who travelled to China at the end of the fourth century AD, was the author of a commentary on:

(a) Prajnaparamita Sutra
(b) Visuddhimagga
(c) Sarvastivada Vinaya
(d)   Lalitavistara
Answer(c)
ExplanationVinaya texts belong to different traditions of Buddhism. Sarvastivadin Vinaya has been preserved in Sanskrit as well as Chinese and Tibetan fragments. At the end of 4th century AD (381-386 AD) Sanghabhuti : the buddhist monk travelled to China . Sanghabhuti is the author of a commentary on the Sarvastivada Vinaya.
 More Information
Reason/ Motivation of questionImportant authors and their contributions
Primary Source of Info.Upindra Singh-History of Ancient and Early Medieval India
Nature of questionStatic
Difficulty levelDifficult
Qn. No. 15
SubjectHistory & Culture
QuestionConsider the following properties included in the World Heritage List released by UNESCO:

1. Shantiniketan.
2. Rani-ki-Vav
3.Sacred Ensembles of the Hoysalas
4. Mahabodhi Temple Complex at Bodhgaya

How many of the above properties were included in 2023?

(a) Only one
(b)  Only two
(c)  Only three
(d)  All four
Answer(b)
ExplanationIndian properties included in the World Heritage List released by UNESCO in 2023 are – Shantiniketan
Sacred Ensembles of the Hoysalas Other sites- Rani-ki-Vav (the Queen’s Stepwell) at Patan, Gujarat (2014)
Mahabodhi Temple Complex at Bodh Gaya 
(2002) (https://whc.unesco.org/en/statesparties/in)
More Information
Reason/ Motivation of questionContemporary issue 
Primary Source of Info.Focus, Compass, The Hindu
Nature of questionCurrent
Difficulty levelModerate

Qn. No. 16
SubjectPolity
QuestionAs per Article 368 of the Constitution of India, the Parliament may amend any provision of the Constitution by way of:

1. Addition
2. Variation
3. Repeal Select the correct answer using the code given below:

(a)  1 and 2 only
(b)  2 and 3 only
(c)  1 and 3 only
(d)  1, 2 and 3
Answer(d)
Explanation368. [Power of Parliament to amend the Constitution and procedure therefor.] Notwithstanding anything in this Constitution, Parliament may in exercise of its constituent power amend by way of addition, variation or repeal any provision of this Constitution in accordance with the procedure laid down in this article.]
More Information
Reason/ Motivation of questionArticle 370 judgment, the Parliament can abrogate Article 370 using its amending powers under Article 368
Primary Source of Info.https://main.sci.gov.in/pdf/LU/article_370.pdf
Nature of questionFactual
Difficulty levelEasy

Qn. No. 17
SubjectInternational Relations
QuestionConsider the following countries:

1.   Italy
2.   Japan
3.   Nigeria
4.   South Korea
5.   South Africa

Which of the above countries are frequently mentioned in the media for their low birth rates, or ageing population or declining population?

(a)   1, 2 and 4
(b)  1, 3 and 5
(c)   2 and 4 only
(d)  3 and 5 only
Answer(a)
ExplanationMedian Age of Population of Japan is 49.5. Median Age of Population of Italy is 48.1. Median Age of Population of South Korea is 45. The above countries are experiencing a rapidly ageing population and also sometimes reducing population.  

While the Median Age of Population of South Africa is 30 and that of Nigeria is 19.1. These countries are experiencing rapidly rising population.
More Information
Reason/ Motivation of questionThese countries experience heavy fatalities during the Covid-19 Pandemic due to their large Old Age population
Primary Source of Info.https://worldpopulationreview.com/country-rankings/median-age
Nature of questionFactual
Difficulty levelModerate
Qn. No. 18
Subject Indian Polity
QuestionWhich of the following statements are correct in respect of a Money Bill in the Parliament?

1. Article 109 mentions special procedure in respect of Money Bills.
2. A Money Bill shall not be introduced in the Council of States.
3. The Rajya Sabha can either approve the Bill or suggest changes but cannot reject it.
4. Amendments to a Money Bill suggested by the Rajya Sabha have to be accepted by the Lok Sabha.

Select the answer using the code given below:

(a)   l and 2only
(b)   2 and 3 only
(c)   1, 2 and 3
(d)   1,3 and 4
Answer(c)
ExplanationStatement 1 is correct as Article 109 is marginally headed as Special procedure in respect of Money Bills.
Statement 2 is correct as Article 109(1) mentions that Money Bill shall not be introduced in the Council of States.
Statement 3 is correct as Article 109((2)) after a Money Bill has been passed by the House of the People it shall be transmitted to the Council of States for its recommendations and the Council of States shall within a period of fourteen days from the date of its receipt of the Bill return the Bill to the House of the People with its recommendations and the House of the People may thereupon either accept or reject all or any of the recommendations of the Council of States.
Statement 4 is INCORRECT as 109(4) If the House of the People does not accept any of the recommendations of the Council of States, the Money Bill shall be deemed to have been passed by both Houses in the form in which it was passed by the House of the People without any of the amendments recommended by the Council of States.
More Information
Reason/ Motivation of question  Aadhar Card judgment
Primary Source of Info.Constitution of India
Nature of questionApplied
Difficulty levelEasy
Qn. No. 19
SubjectInternational Relations
QuestionWhich of the following is/are correctly matched in terms of equivalent rank in three services of Indian Defence forces?
S.N. Army Airforce Navy
1. Brigadier Air Commodore Commander
2. Major General Air Vice Marshal Vice Admiral
3. Major Squadron Leader Lieutenant Commander
4. Lieutenant Colonel Group Captain Captain

Select the correct answer using the code given below:

(a)  1 and 4
(b)  1 and 3
(c)  2, 3 and 4
(d)  3 only
Answer(d)
 Explanation S.N. Army Airforce navy 1. Brigadier Air Commodore Commodore 2. Major General Air Vice Marshal Rear Admiral 3. Major Squadron Leader Lieutenant Commander 4. Lieutenant Colonel Wing Commander Commander
 More Information
Reason/ Motivation of question Defence/Security
Primary Source of Info.https://sansad.in/getFile/loksabhaquestions/annex/ 178/AU474.pdf?source=pqals
Nature of questionStatic
Difficulty levelDifficult
Qn. No. 20
SubjectIndian Polity
QuestionThe North Eastern Council (NEC) was established by the North Eastern Council Act, 1971. Subsequent to the amendment of NEC Act in 2002, the Council comprises which of the following members?

1. Governor of the Constituent State
2.Chief Minister of the Constituent State
3.Three Members to be nominated by the President of India
4. The Home Minister of India

Select the correct answer using the code given below:

(a)   1, 2 and 3 only
(b)   1, 3 and 4 only
(c)   2 and 4 only
(d)   1, 2, 3 and 4
Answer(a)
ExplanationNorth Eastern Council Act, 1971 was amended in the year 2002 and it amended Section 3 of the Principal Act of 1971. The 2002 Amendment can be found here: https://necouncil.gov.in/sites/default/files/about-us/Amendment%20Act%202002.pdf The North East Council Act was Amended in 2012, Section 3 of the Act was amended and 3(1) was added, which read; “3(1) There shall be a Council to be called the North Eastern Council which shall consist of the following members, namely:-

(i) the person or persons for the time being holding the office of the Governor of the States; (STATEMENT 1)

(ii) the Chief Ministers of the States of Arunachal Pradesh, Assam, Manipur, Meghalaya, Mizoram, Nagaland Sikkim and Tripura;  (STATEMENT 2) Provided that, if there is no Council of Ministers in any state referred to in clause (b), the President may nominate not more than one person to represent such State in the Council for so long as there is no Council of Ministers in such State;

(ii) three members to be nominated by the President”:  (STATEMENT 3) (b) for sub-section (3), the following sub-section shall be substituted namely:- (3) the President shall nominate the Chairman of the Council”. Based on this, the Union Cabinet chaired by the Prime Minister approved the proposal of Ministry of Development of North Eastern Region (DoNER) for the nomination of the Union Home Minister as ex-officio Chairman of North Eastern Council (NEC) – a statutory body with Governors and Chief Ministers of all the eight North Eastern States as its Member. The Cabinet also approved that the Minister of State (Independent Charge), Ministry of DoNER would serve as Vice Chairman of the Council. (STATEMENT 4) https://pib.gov.in/newsite/PrintRelease.aspx?relid=179941 https://necouncil.gov.in/sites/default/files/about-us/Amendment%20Act%202002.pdf
 More Information
Reason/ Motivation of questionStatic portion of Polity, Various constitutional and statutory bodies.
Primary Source of Info.https://necouncil.gov.in/sites/default/files/about-us/Amendment%20Act%202002.pdf

https://pib.gov.in/newsite/PrintRelease.aspx?relid=179941

https://www.indiacode.nic.in/bitstream/123456789/1580/2/A1971-84.pdf
Nature of questionFactual
Difficulty levelDifficult
Qn. No. 21
SubjectIndian Polity
QuestionConsider the following statements regarding ‘Nari Shakti Vandan Adhiniyam’:

1. Provisions will come into effect from the 18th Lok Sabha.
2. This will be in force for becoming an Act.
3. There are provisions for the reservation of seats for Scheduled Castes Women within the quota reserved for the Scheduled Castes.

Which of the statements given above are correct?

(a) 1, 2 and 3
(b)  1 and 2 only
(c)   2 and 3 only
(d)  1 and 3 only
Answer(c)
ExplanationStatement 1 is INCORRECT: The provisions shall come into effect after an exercise of delimitation is undertaken for this purpose after the relevant figures for the first census taken after commencement of the Constitution (One Hundred and Sixth Amendment) Act, 2023 have been published.

Statement 2 is CORRECT:  Provisions of Act shall cease to have effect on the expiration of a period of “fifteen years from such commencement”

Statement 3  is Correct: Section 3(2) of the Act states that  as nearly as may be, one-third of the total number of seats reserved under clause (2) of article 330 shall be reserved for women belonging to the Scheduled Castes or the Scheduled Tribes.
More Information
Reason/Motivation of question Current Affairs (Amendments to the Constitution)
 Primary Source of Info.https://egazette.gov.in/WriteReadData/2023/249053.pdf
Nature of questionFactual Current Affairs
 Difficulty levelModerate
 Qn. No. 22
SubjectInternational Relations
QuestionWhich of the following statements about ‘Exercise Mitra Shakti-2023’ are correct?
1.    This was a joint military exercise between India and Bangladesh
2.    It commenced in Aundh (Pune).
3.    Joint response during counter-terrorism operations was a goal of this operation. 4.    Indian Air Force was a part of this exercise.

Select the answer using the code given below:

(a)   1, 2 and 3
(b)   1, 2 and 4
(c)   1, 3 and 4
(d)   2, 3 and 4
Answer(d)
Explanation Statement 1 is wrong as MITRA SHAKTI is a JOINT EXERCISE between  INDIA – SRI LANKA
 More Information
Reason/ Motivation of question India – Sri Lanka ties
Primary Source of Info.https://pib.gov.in/PressReleaseIframePage.aspx? PRID=1977339
Nature of questionCurrent affairs
 Difficulty levelEasy
Qn. No. 23
SubjectIndian Polity
QuestionA Writ of Prohibition is an order issued by the Supreme Court or High Courts to:

(a)   a government officer prohibiting from taking a particular action.
(b)   the Parliament / Legislative Assembly to pass a law on Prohibition.
(c)   the lower court prohibiting continuation of proceedings in a case.
(d)   the Government prohibiting it from following an unconstitutional policy.
Answer(c)
ExplanationProhibition: This writ is issued by a higher court (High Court or Supreme Court) when a lower court has considered a case going beyond its jurisdiction. Writs of Certiorari and Prohibition have much similarity in as much as both are issued by a higher court against an inferior court or tribunal or any authority having judicial or quasi-judicial functions and the object of both the writs is to see that inferior courts or tribunals act within their jurisdiction strictly in accordance with the legal rules. The two writs, however, differ in one respect as they are issued at different stages of the proceeding. Where the inferior court has not decided the matter but is going to decide, writ of prohibition will be issued to stop the proceedings but where the matter has already been decided by the inferior court or tribunal, the writ of certiorari will be issued to quash the decision.
More Information
Reason/ Motivation of question Basics of Indian Polity
Primary Source of Info.https://ncert.nic.in/ncerts/l/keps202.pdf (page 41)
Nature of questionFactual
Difficulty levelEasy
Qn. No. 24
SubjectIndian Polity
QuestionConsider the following statements:
1.    It is the Governor of the State who recognizes and declares any community of that State as a Scheduled Tribe.
2.    A community declared as a Scheduled Tribe in a State need not be so in another State.

Which of the statements given above is/are correct?

(a)   1 only (b)   2 only (c)   Both 1 and 2 (d)   Neither 1 nor 2
Answer(b)
Explanation Article 342(1) provides that the President may with respect to any State or Union Territory, and where it is a State, after consultation with the Governor thereof, by a public notification, specify the tribes or tribal communities or part of or groups within tribes or tribal communities as Scheduled Tribe in relation to that State or Union Territory as the case may be. While the Constitution is silent about the criteria for specification of a community as a Scheduled Tribe.

The words and the phrase ‘tribes or tribal communities or part of or groups within tribes or tribal communities” in Article 342 have to be understood in terms of their historical background of backwardness. Primitiveness, geographical isolation, shyness and social, educational & economic backwardness due to these reasons are the traits that distinguish Scheduled Tribe communities of our country from other communities. It takes into account the definitions of tribal Communities adopted in the 1931 Census. These facts are the basis for the provision in Article 342(1) which mandates to specify the tribes or tribal communities or part of or groups within tribes or tribal communities as Scheduled Tribe in relation to that State or Union Territory as the case may be.

Thus the list of Scheduled Tribes is State/UT specific and a community declared as a Scheduled Tribe in a State need not be so in another State.
More Information
Reason/ Motivation of question The Union Cabinet approved the addition of four tribes to the list of Scheduled Tribes. The Hatti tribe in the Trans-Giri area of Sirmour district in Himachal Pradesh, the Narikoravan and Kurivikkaran hill tribes of Tamil Nadu and the Binjhia in Chhattisgarh, who were listed as ST in Jharkhand and Odisha but not in Chhattisgarh, were the communities newly added to the list.
Primary Source of Info.https://ncst.nic.in/content/frequently-asked-questions#:~:text=342(1)%20Scheduled%20Tribes%20%2D%2D%2D, Scheduled%20Tribe%20in%20relation%20to
Nature of questionApplied
Difficulty levelModerate
Qn. No. 25
SubjectIndian Polity
QuestionWith reference to Union Budget, consider the following statements:

1. The Union Finance Minister on behalf of the Prime Minister lays the Annual Financial Statement before both the Houses of Parliament.

2. At the Union level, no demand for a grant can be made except on the recommendation of the Provident of lndia.

Which of the statements given above is/are correct?


(a)   1 only
(b)   2 only
(c)   Both land 2
(d)   Neither 1 nor 2
Answer(b)
 Explanation STATEMENT 1 – Is INCORRECT because of Article 112 which states: The President shall in respect of every financial year cause to be laid before both the Houses of Parliament a statement of the estimated receipts and expenditure of the Government of India for that year, in this Part referred to as the “annual financial statement”. Thus, the Annual Financial Statement is presented on behalf of the President and not the Prime Minister.

STATEMENT 2 – is CORRECT because of Article 113(3) which states: No demand for a grant shall be made except on the recommendation of the President.
More Information
Reason/ Motivation of question Testing the basic understanding of students on “PARLIAMENTARY PROCESS INCLUDING THE ANNUAL FINANCIAL STATEMENT i.e. The Budget ”
Primary Source of Info.Constitution of India – Bare Act
Nature of questionFactual from the Constitution
Difficulty levelEasy
Qn. No. 26
SubjectInternational Relations
QuestionWho of the following is the author of the books “The India Way” and “Why Bharat Matters”?

(a)   Bhupender Yadav
(b)   Nalin Mehta
(c)   Shashi Tharoor
(d)   Subrahmanyam Jaishankar
Answer(d)
 Explanation Both the books have been written by S. Jaishankar
More Information
Reason/ Motivation of question Recently written by Minister of External Affairs
 Primary Source of Info.https://economictimes.indiatimes.com/news/india/important-to-understand-why-bharat-matters-because-eam-jaishankar/articleshow/107957013.cms?from=mdr
Nature of questionCurrent affairs
 Difficulty levelEasy
Qn. No. 27
SubjectInternational Relations
QuestionConsider the following pairs:

S. No. Country Reason for being in the news
1. Argentina Worst economic crisis
2. Sudan War between the country’s regular army and paramilitary forces
3. Turkey Rescinded its membership of NATO

How many of the pairs given above are correctly matched?

(a)  Only one pair
(b) Only two pairs
(c)  All three pair
(d)  None of the pairs
Answer(b)
Explanation Turkey has not rescinded is membership of NATO
More Information
Reason/ Motivation of question Countries in News
 Primary Source of Info.QIP Classes plus IR compass
Nature of questionCurrent affairs
 Difficulty levelEasy
Qn. No. 28
SubjectInternational Relations
QuestionConsider the following statements:
Statement-I: Sumed pipeline is a strategic route for Persian Gulf oil and natural gas shipment to Europe

Statement-II: Sumed pipeline connects the Red Sea with the Mediterranean Sea. Which one of the following is correct in respect of the above statements? (a)   Both Statement-I and Statement-II are correct and Statement-II explains Statement-I (b)   Both Statement-I and Statement-II are correct, but Statement-II does not explain Statement-I (c)   Statement-I is correct, but Statement-II is incorrect (d)   Statement-I is incorrect, but Statement-II is correct
Answer(a)
Explanation The Sumed Pipeline (also known as the Suez-Mediterranean Pipeline) is an oil pipeline in Egypt, running from the Ain Sokhna terminal in the Gulf of Suez, the northernmost terminus of the Red Sea, to offshore Sidi Kerir port, Alexandria in the Mediterranean Sea. It provides an alternative to the Suez Canal for transporting oil from the Persian Gulf region to the Mediterranean.
More Information
Reason/ Motivation of question Countries in News
 Primary Source of Info.https://www.eia.gov/todayinenergy/detail.php?id=40152
Nature of questionCurrent affairs
 Difficulty levelDifficult
Qn. No. 29
SubjectGeography
QuestionConsider the following statements:

1.  The Red Sea receives very precipitation in any form.
2. water enters the Red Sea from rivers.

Which of the statements given above is/are correct?

(a)   1 only
(b)   2 only
(c)   Both 1 and 2
(d)   Neither 1 nor 2
Answer(a)
Explanation Red sea lies in sub Tropical area and is bordered by vast dry land masses and hence does not receive much rainfall. Though it receives many seasonal rivers like River Hadas, Anseba, Barka etc
More Information
 Reason/Motivation of question Red sea area conflict in news
 Primary Source of Info.Classroom program
Nature of questionCurrent affair and Basic
 Difficulty levelDifficult
Qn. No. 30
SubjectEnvironment
QuestionAccording to the Environmental Protection Agency (EPA), which one of the following is the largest source of sulphur dioxide emissions?

(a)   Locomotives using fossil fuels
(b)   Ships lining fossil fuels
(c)   Extraction of metals from ores
(d)   Power plants using fossil fuels
Answer(d)
Explanation According to the EPA’s official website: EPA’s national ambient air quality standards for SO2 are designed to protect against exposure to the entire group of sulfur oxides (SOx).  SO2 is the component of greatest concern and is used as the indicator for the larger group of gaseous sulfur oxides (SOx).  Other gaseous SOx (such as SO3) are found in the atmosphere at concentrations much lower than SO2. Emissions that lead to high concentrations of SO2 generally also lead to the formation of other SOx. The largest sources of SO2 emissions are from fossil fuel combustion at power plants and other industrial facilities. Smaller sources of SO2 emissions include: industrial processes such as extracting metal from ore; natural sources such as volcanoes; and locomotives, ships and other vehicles and heavy equipment that burn fuel with a high sulfur content.   
More Information
 Reason/Motivation of question Climate Change & Air Pollution / Pollutants – Recurring Theme
Primary Source of Info.Rau’s IAS Foundation Magazine (Environment)   https://www.epa.gov/so2-pollution/sulfur-dioxide-basics#:~:text=The%20largest%20source%20of%20SO,plants%20and%20other%20industrial%20facilities.
Nature of questionFactual
 Difficulty levelEasy
Qn. No. 31
SubjectEconomy
QuestionConsider the following statements:

Statement-I: If the United States of America (USA) were to default on its debt, holders of US Treasury Bonds will not be able to exercise their claims to receive payment.

Statement-II: The USA Government debt is not backed by any hard assets, but only by the faith of the Government. Which one of the following is correct in respect of the above statements?

(a)  Both Statement-I and Statement-II are correct and Statement-II explains Statement-I.

(b)  Both Statement-I and Statement-II are correct, but Statement-II does explain.

(c)  Statement-I is correct, but Statement-II is incorrect.

(d)  Statement-I is incorrect, but Statement-II is correct.
Answer(a)
ExplanationWhen the Government issues Bonds to borrow money, people lend their money based upon the faith that the Government would repay the borrowed money. The Bonds are backed by the Sovereign promise of the Government that it would repay the money. The Bonds are not backed by any hard assets.


Hence, Statement 2 is correct. So, the question which arises is what would happen if the Government defaults on the Bond Payment. Can People exercise their claims to receive payment? Here, you need to understand what it means to “Exercise their claims to receive payments?”. “Exercising claims” means that the Bond holders are entitled under a law to sue the Government to receive their payments. For example, In case of Default by the corporates, the Bond Holders would have a right to exercise their claims under Bankruptcy law (For example, IBC in India). However, there is no such law that would enable Bond holders to exercise their claims against the Government as it enjoys Sovereign immunity. Further, in the case of Corporates, the Bond holders can exercise their claims to receive the payments by selling off the assets of the companies. However, in case of default by the Government, the Bond holders cannot sell off the assets of the Government. Hence, to the best of our understanding,

Statement 1 is correct. Statement 2 is the correct explanation of Statement 1.
Hence, the correct answer to the question is option A.
More Information
Reason/ Motivation of questionThe Question has been asked in the context of the US Debt Ceiling Crisis.
Primary Source of Info. 
Nature of questionCurrent Affairs (Public Finance)
Difficulty levelMedium
Qn. No. 32
SubjectEconomy
Question Consider the following statements:

Statement-I: Syndicated lending spreads the risk of borrower default across multiple lenders.

Statement-II: The syndicated loan can be a fixed amount/lump sum of funds, but cannot be a credit line. Which one of the following is correct in respect of the above statements?

(a)   Both Statement-I and Statement-II are correct and Statement-II explains Statement-I.

(b)  Both Statement-I and Statement-II are correct, but Statement-II does explain.

(c)   Statement-I is correct, but Statement-II is incorrect.

(d)  Statement-I is incorrect, but Statement-II is correct.
Answer(c)
ExplanationMeaning: Banks provide large value loans to the MNCs. In certain cases, a single Bank may not be able to lend large value loans. Hence, in such cases, two or more Banks can enter into a loan agreement and lend money to MNC. Such an agreement is called a Syndicate Loan agreement.

Mechanism: The borrower selects a bank or financial institution to act as a nodal agent for syndication which then invites participation of other banks and financial institutions to finance the single borrower. Although the borrower signs a common document, drawn up by the syndicate manager, the borrower has a distinct contractual relationship with each of the syndicate members.

Type of Loans: The loan may include a fixed amount of funds, line of credit (continuous transfer of amounts over a period), or a combination of the two.

Advantages: Enable MNCs to avail large value loansRisk of default by the borrower is spread across multiple lenders.

Hence, Statement 1 is correct.Loans can be provided both in the form of lump sum funds or line of Credit. Hence, Statement 2 is incorrect. RBI’s Guidelines: Subject to the RBI regulatory norms, the parties are free to structure the syndicated lending as per their requirements. RBI has also directed all banks engaged in syndicated lending to regularly share the credit information of respective borrowers with one another and Credit Information companies to ensure transparency and reduce fraud among lenders.
More Information
Reason/ Motivation of questionIn 2021, the RBI proposed to revise guidelines related to Syndicated Lending.
Primary Source of Info.https://blog.ipleaders.in/understanding-syndicate-loan-agreement/  https://m.rbi.org.in/scripts/BS_CircularIndexDisplay.aspx?Id=3705
Nature of questionCurrent Affairs (Banking & Finance)
Difficulty levelDifficult

Qn. No. 33
SubjectEconomy
QuestionConsider the following statements in respect of the digital rupee:

1.   It is sovereign currency issued by the Reserve Bank of India (RBI) in alignment with its monetary policy.
2.   It appears as a liability on the RBI’s balance sheet.
3.   It is insured against inflation by its very design.
4.   It is freely convertible against commercial bank money and cash.

Which of the statements given above are correct?
(a)   1 and 2 only
(b)  1 and 3 only
(c)   2 and 4 only
(d)  1, 2 and 4
Answer(d)
ExplanationExplanation: Details about Digital Rupee The Digital Rupee (CBDC) has been introduced through an amendment to the RBI Act, 1934 based upon the recommendations of the Subhash Chandra Garg Committee.

Hence, Statement 1 is correct. Just like Bank note, the CBDC is a liability of the RBI. Hence,
Statement 2 is correct. The CBDC is not insured against Inflation.
Hence, Statement 3 is incorrect. The CBDC is fungible i.e., it can be converted into Deposits and Cash.
Hence, Statement 4 is correct.
 More Information
Reason/ Motivation of questionIn 2022-23, the RBI introduced CBDC to promote cashless economy and facilitate digital payments.
Primary Source of Info.https://rbi.org.in/Scripts/PublicationReportDetails.aspx? UrlPage=&ID=1218
Nature of questionCurrent Affairs (Banking & Finance)
Difficulty levelEasy
Qn. No. 34
SubjectHistory & Culture
QuestionWith reference to ancient India. Gautama Buddha was generally known by which of the following epithets?

1.   Nayaputta
2.   Shakyamuni
3.   Tathagata

Select the correct answer using the code given below:

(a)   1 only
(b)  2 and 3 only
(c)   1, 2 and 3
(d)   None of the above are epithets of Gautama Buddha
Answer(b)
ExplanationGautam Buddha was the founder of Buddhist philosophy which became religion later. Reigning between 563 BCE or 480 BCE, he was known by various names and titles including Sakyamuni (sage of the Sakya clan), Tathagata (the one “thus-gone” or “thus-come”), Cakkhuma, Bhisakka, Angirasa etc. While various surviving early Jain and Buddhist literature uses several names (or epithets) for Mahavira, including NayaputtaMuniSamanaNiganthaBrahman, and Bhagavan.
More Information
Reason/ Motivation of questionBuddhism and Jainism are one of the most repeated topics in the UPSC prelims examination.
Primary Source of Info.Upinder Singh – A history of Ancient and Early Medieval India
https://en.wikipedia.org/wiki/Mahavira
Nature of questionStatic
 Difficulty levelModerate
 Qn. No. 35
SubjectHistory & Culture
Question Consider the following information:

S.N. Archaeological Site State Description
1. Chandrakotugarh Odisha Trading Port town
2. Inamgaon Maharashtra Chalcolithic site
3. Mangadu Kerala Mogalithic site
4. Salihundam Andhra Pradesh Rock-cut cave shrines

In which of the above rows is the given information correctly matched?

(a)   1 and 2 
(b)   2 and 3
(c)   3 and 4 
(d)   1 and 4
Answer(b)
Explanation Archeological site State Description 1. Chandraketugard West Bengal Trading port town 2. Inamgaon Maharashtra Chalcolithic site 3. Mangadu Kerala Megalithic site 4. Salihundam Andhra Pradesh Bricks based Buddhist Stupa
More Information
Reason/ Motivation of question  Buddhism and Jainism are one of the most repeated topics in the UPSC prelims examination. Map based questions are frequently being asked in the past few prelims by the UPSC.
Primary Source of Info.NCERT- R S Sharma (India’s Ancient Past)NCERT- Class 6th, Our Past-IRau’s IAS Compass of History and CultureA history of Ancient and Early Medieval India – Upinder Singh
Nature of questionStatic
Difficulty levelModerate
 Qn. No. 36
SubjectHistory & Culture
Question Who of the following rulers of medieval India gave permission to the Portuguese to build a fort at Bhatkal?

(a)  Krishnadevaraya

(b)  Narasuiiha Saluva

(c)  Muhammad Shall III

(d)  Yusuf Adil Shah
Answer(a)
ExplanationAs per a treaty signed with the Portuguese in the year 1510, Sri Krishnadeva Raya permitted Albuquerque (Portuguese governor) to build a fort at Bhatkal.
 More Information
Reason/ Motivation of question  Vijayanagar is the most sought after topic for UPSC prelims examination.  
Primary Source of Info.NCERT Map from 12th class History NCERT (Theme II)
Nature of questionStatic
Difficulty levelModerate  
 Qn. No. 37
SubjectHistory & Culture
Question With reference to revenue collection by Cornwallis, statements: consider the following

1.   Under the Ryotwari Settlement of revenue collection, the peasants were exempted from revenue payment in case of bad harvests or natural calamities.

2.   Under the Permanent Settlement in Bengal, if the Zamindar failed to pay his revenues to the state, on or before the fixed date, he would be removed from his Zamindari.

Which of the statements given above is/are correct?

(a)   1 only
(b)   2 only
(c)   Both 1 and 2
(d)   Neither 1 nor 2
Answer(b)
Explanation1st statement is wrong. Ryotwari settlement was not brought by Cornwallis. Also, even in Ryotwari settlement, the farmers were not exempted from payment of revenues. 2nd Statement is correct. In case of failure of the Zamindar to pay the revenue in time, under Sunset Law, Zamindar would be removed from Zamindari.
 More Information
Reason/ Motivation of questionImportant theme of Modern Indian history
Primary Source of Info.Bipan Chandra: Old NCERT.
Nature of questionStatic
Difficulty levelModerate
Qn. No. 38
SubjectHistory & Culture
 QuestionConsider the following statements:

1.   There are no parables in Upanishads.

2.   Upanishads were composed earlier than the Puranas.

Which of the statements given above is/are correct?

(a)   1 only
(b)   2 only
(c)   Both 1 and 2
(d)   Neither 1 nor 2
Answer(b)
Explanation1st statement is wrong. There are many parables in Upanishads, like Kathopnishad has the story of Yama and Nahciketa. 2nd statement is correct. Upanishads were compiled by the end of Later Vedic Age, Puranas were finally compiled by the Gupta Age.
More Information
Reason/ Motivation of questionImportant theme in Ancient India.
Primary Source of Info.R S Sharma: Old NCERT
Nature of questionStatic
Difficulty levelModerate
Qn. No. 39
SubjectInternational Relations
Question Consider the following statements:

1.   India is a member of the International Grains Council.
2.   A country needs to be a member of (lie International Grains Council for exporting or importing rice and wheat.

Which of the statements given above is/are correct?

(a)   1 only
(b)  2 only
(c)   Both 1 and 2
(d)  Neither 1 nor 2
Answer(a)
ExplanationStatement 2 is wrong: A country needs not to be a member of the International Grains Council for exporting or importing rice and wheat.
More Information
Reason/ Motivation of questionIndia announced a ban on export of non-basmati rice
Primary Source of Info.https://compass.rauias.com/current-affairs/international-grains-council/
Nature of questionCurrent affairs
Difficulty levelDifficult
 Qn. No. 40
SubjectHistory & Culture
Question Which one of the following was the latest inclusion in the Intangible Cultural Heritage List of UNESCO?

(a)   Chhau dance
(b)   Durga puja
(c)   Garba dance
(d)   Kumbh mela
Answer(c)
 ExplanationThe latest addition in the list of Intangible Cultural Heritage of UNESCO is Garba dance.  
 More Information
Reason/ Motivation of question  Contemporary issue
Primary Source of Info.The Hindu/ Focus
Nature of questionContemporary
Difficulty levelModerate
Qn. No. 41
SubjectInternational Relations
QuestionConsider the following statements:
Statement-I: There is instability and worsening security situation in the Sahel region.
Statement-II: There have been military takeovers/coups d’état in several countries of the Sahel region in the recent past.

Which one of the following is correct in respect of the above statements?

(a)   Both Statement-I and Statement-II are correct and Statement-II explains Statement-I
(b)   Both Statement-I and Statement-II are correct, but Statement-II does not explain Statement-I
(c)   Statement-I is correct, but Statement-II is incorrect
(d)   Statement-I is incorrect, but Statement-II is correct
Answer(a)
 ExplanationBoth the statements are correct and the second statement clearly explains the reason for instability.
 More Information
Reason/ Motivation of questionIndia announced a ban on export of non-basmati rice
Primary Source of Info.QIP classes, Rau’s Prelims compass and IAS compass website https://compass.rauias.com/current-affairs/global-terrorism-index/
Nature of questionCurrent affairs
Difficulty levelModerate
 Qn. No. 42
SubjectEconomy
QuestionConsider the following statements:
Statement-I: India does not import apples from the United States of America.
Statement-II: In India, the law prohibits the import of Genetically Modified food without the approval of the competent authority.

Which one of the following is correct in respect of the above statements?

(a)   Both Statement-I and Statement-II are correct and Statement-1 Statement-II explains
(b)   Both Statement-I and Statement-II are correct, but Statement-II does not explain Statement-I
(c)   Statement-I is correct, but Statement-II is incorrect
(d)   Statement-I is incorrect, but Statement-II is correct
Answer(d)
 ExplanationIndia imports Apples from the US. Hence, Statement 1 is incorrect.
In India, the Food Safety and Standards Act, 2006 prohibits import, manufacture or sale of Genetically Modified Food without the approval of FSSAI. Hence. Statement 2 is correct.
More Information
Reason/ Motivation of question In 2019, the US Government decided to withdraw Trade privileges given to India in the form of the Generalised System of Preferences (GSP). In response, India decided to impose retaliatory tariffs on the import of certain goods from the USA, including Apples. In 2023, India and USA mutually agreed to resolve their trade disputes wherein India agreed to reduce tariffs on import of certain goods from USA, including Apples.
Primary Source of Info.https://pib.gov.in/PressReleseDetailm.aspx?PRID=1935460 https://pib.gov.in/newsite/PrintRelease.aspx?relid=147678 #:~:text=Import%20of%20food%20products%20is%20regulated %20under,after%20necessary%20approval/No%20Objection%20 Certificate(NOC)%20by%20FSSAI.
Nature of questionCurrent Affairs (External Sector)
Difficulty levelMedium

Qn. No. 43
SubjectIndian Polity
QuestionWith reference to the Speaker of the Lok Sabha, consider the following statements:
While any resolution for the removal of the Speaker of the Lok Sabha is under consideration
1.    He/She shall not preside.
2.    He/She shall not have the right to speak
3.    He/She shall not be entitled to vote on the resolution in the first instance.

Which of the statements given above is/are correct?

(a)   1 only
(b)   1 and 2 only
(c)   2 and 3 only
(d)   1, 2 and 3
Answer(a)
Explanation The Answer for all the statements can be derived from a basic understanding of Article 96 of the Constitution of India.

Article 96 mentions about the Speaker or the Deputy Speaker not to preside while a resolution for his removal from office is under consideration.

Article 96(1) – At any sitting of the House of the People, while any resolution for the removal of the Speaker from his office is under consideration, the Speaker, or while any resolution for the removal of the Deputy Speaker from his office is under consideration, the Deputy Speaker, shall not, though he is present, preside (Statement-1), and the provisions of clause (2) of article 95 shall apply in relation to every such sitting as they apply in relation to a sitting from which the Speaker, or, as the case may be, the Deputy Speaker, is absent.

Article 96(2) – The Speaker shall have the right to speak in (Statement-2), and otherwise to take part in the proceedings of, the House of the People while any resolution for his removal from office is under consideration in the House and shall, notwithstanding anything in article 100, be entitled to vote only in the first instance (Statement-3) on such resolution or on any other matter during such proceedings but not in the case of an equality of votes. However, since the Home Minister was not added to the North-Eastern Council due to the NEC Amendment Act, 2002. Hence, this is not correct. Thus, based on the understanding of Article 96, we can conclude that ONLY STATEMENT 1 is CORRECT.
  More Information
Reason/ Motivation of questionTesting the basic understanding of students on “POWERS OF SPEAKER INCLUDING HIS REMOVAL” from the constitutional provisions.
Primary Source of Info.CONSTITUTION OF INDIA – BARE ACT
Nature of questionFACTUAL
Difficulty levelMODERATE
Qn. No. 44
SubjectIndian Polity
QuestionWith reference to the Indian Parliament, consider the following statements:

1.    A bill pending in the Lok Sabha lapses on its dissolution.
2.    A bill passed by the Lok Sabha and pending in the Rajya Sabha lapses on the dissolution of the Lok Sabha.
3.    A bill in regard to which the President of India notified his/her intention to summon the Houses to a joint sitting lapses on Lok Sabha. the dissolution of the Loksabha.

Which of the statements given above is/are correct?

(a)   1 only
(b)   1 and 2
(c)   2 and 3
(d)   3 only
Answer(b)
ExplanationWhen Does The Bill Lapses When the Bill Does Not Lapse Bill pending in Lok Sabha   Bill passed by Lok Sabha, pending in Rajya Sabha Bill pending in Rajya Sabha but not passed by Lok Sabha Bill passed by both Houses but awaited President’s Assent Bill passed by both Houses but returned by President for reconsideration This basic understanding can be found under Article 107(3) & (4) and Article 108(5).  

Article 107 (3) – A Bill pending in Parliament shall not lapse by reason of the prorogation of the Houses. 

Article 107(4) – A Bill pending in the Council of States which has not been passed by the House of the People shall not lapse on a dissolution of the House of the People. Article 108(5): A joint sitting may be held under this article and a Bill passed thereat, notwithstanding that a dissolution of the House of the People has intervened since the President notified his intention to summon the Houses to meet therein. Thus, a Bill shall not lapse once it has been notified for Joint Sitting by the President.
More Information
Reason/ Motivation of question Testing the basic understanding of students on “PARLIAMENTARY PROCESS ON PASSING OF BILL AND LAPSING OF BILL ON DISSOLUTION OF LOK SABHA”. This can be easily understood through Article 107. In 2024,the 17th Lok Sabha was dissolved and 18th Lok Sabha was constituted.
Primary Source of Info.CONSTITUTION OF INDIA & LAXMIKANTH
Nature of questionFactual
Difficulty levelEasy
Qn. No. 45
SubjectIndian Polity
QuestionWith reference to the Parliament of India, consider the following statements:

1.    Prorogation of a House by the President of India does not require the advice of the Council of Ministers.
2.    Prorogation of a House is generally done after the House is adjourned sine die but there is no bar to the President of India prorogating the House which is in session.
3.    Dissolution of the Lok Sabha is done by the President of India who, save in exceptional circumstances, does so on the advice of the Council of Ministers

Which of the statements given above is/are correct?

(a)   1 only
(b)   1 and 2
(c)   2 and 3
(d)   3 only
Answer(c)
 Explanation STATEMENT 1 Prorogation of the House under Article 85 is done by the President. This is not the discretionary power of the President and is always done on the advice of Council of Ministers. Hence, Statement 1 is incorrect   Both Statements 2 and 3 are CORRECT STATEMENT 2 Prorogation brings an end to a session of the house done by an order of the President under Article 85(2)(a).

Prorogation of the House may take place any time, even while the House is sitting.  However, usually, prorogation follows the adjournment of the sitting of the House sine die. Prorogation terminates both the sitting and the session of the House Adjournment Sine Die refers to adjournment of the House of Parliament for an indefinite period without assigning any specific day or date for its reconvening. The power of Adjournment Sine Die lies with the presiding officer of the House – Chairman/Speaker.  

STATEMENT 3 – Dissolution of Lok Sabha is generally done on the advice of the Prime Minister and his Council. However, there may be times when the government in power needs to prove majority on the floor of the House and is defeated and it is difficult for any coalition to form a government. In such exceptional situations, the President may dissolve the Lok Sabha. (Article 85 of Constitution)
 More Information
Reason/ Motivation of question  Dissolution of Current Lok Sabha
Primary Source of Info.https://www.mpa.gov.in/sites/default/files/parlia3.pdf
Nature of questionStatic Polity
Difficulty levelModerate
Qn. No. 46
SubjectEnvironment
QuestionConsider the following statements:

Statement-I: The European Parliament approved The Net-Zero Industry Act recently.

Statement-II: The European Union intends to achieve carbon neutrality by 2040 and therefore aims to develop all of its own clean technology by that time. Which one of the following is correct in respect of the above statements?

(a)   Both Statement-I and Statement-II are correct and Statement-I Statement-II explains

(b)   Both Statement-I and Statement-II are correct, but Statement-II does not explain Statement-1

(c)   Statement-I is correct, but Statement-II is incorrect

(d)   Statement-I is incorrect, but Statement-II is correct
Answer(c)
ExplanationStatement 1 is correct – In April’ 2024 , the European Parliament approved the Net-Zero industry Act to bolster EU production in technologies needed for decarbonisation.Statement 2 is incorrect – The Net-Zero Industry Act is an initiative stemming from the Green Deal Industrial Plan which aims to scale up the manufacturing of clean technologies in the EU. The aim is that the Union’s overall strategic net-zero technologies manufacturing capacity approaches or reaches at least 40% of annual deployment needs by 2030. This will accelerate the progress towards the EU’s 2030 climate and energy targets and the transition to climate neutrality by 2050. It will also boost the competitiveness of EU industry, create quality jobs, and support the EU’s efforts to become energy independent.
 More Information
Reason/ Motivation of questionCurrent Affairs & Climate Change
Primary Source of Info.https://www.europarl.europa.eu/news/en/press-room/20240419IPR20568/meps-adopt-plans-to-boost-europe-s-net-zero-technology-production

https://economictimes.indiatimes.com/tech/technology/eu-lawmakers-back-law-to-boost-domestic-clean-tech-production/articleshow/109598885.cms?from=mdr

https://commission.europa.eu/strategy-and-policy/priorities-2019-2024/european-green-deal/green-deal-industrial-plan/net-zero-industry-act_en
Nature of questionFactual
Difficulty levelModerate
Qn. No. 47
SubjectEconomy
QuestionConsider the following statements:

Statement-I: Recently, Venezuela has achieved a rapid recovery from its economic crisis and succeeded in preventing its people from fleeing/emigrating to other countries.

Statement-II: Venezuela has the world’s largest oil reserves. Which one of the following is correct in respect of the above statements?

(a)   Both Statement-I and Statement-II are correct and Statement-II explains Statement-I

(b)   Both Statement-I and Statement-II are correct, but Statement-II does not explain Statement-I

(c)   Statement-I is correct, but Statement-II is incorrect

(d)   Statement-1 is incorrect, but Statement-II is correct
Answer(d)
ExplanationEconomic Crisis in Venezuela in the form of hyperinflation, poverty, lack of access to basic services etc. has forced a large number of people to migrate to other countries. Even though Venezuela has shown signs of Economic Recovery, it is still reeling under an Economic crisis. It has not succeeded in preventing its people from migrating to other countries. Hence, statement 1 is incorrect. Venezuela has the world’s largest proven oil reserves. Hence, statement 2 is correct.
More Information
Reason/ Motivation of questionEconomic Crisis in Venezuela in the form of hyperinflation, poverty, lack of access to basic services etc. has forced a large number of people to migrate to other countries.
Primary Source of Info.https://www.unrefugees.org/news/venezuela-crisis-explained/ https://www.eia.gov/international/analysis/country/VEN
Nature of questionCurrent Affairs (External Sector)
Difficulty levelMedium
Qn. No. 48
SubjectEconomy
QuestionWith reference to the Digital India Land Records Modernisation Programme, consider the following statements:

1.    To implement the scheme, the Central Government provides 100% funding.

2.    Under the Scheme, Cadastral Maps are digitised.

3.    An initiative has been undertaken to transliterate the Records of Rights from local language to any of the languages recognized by the Constitution of India.

Which of the statements given above are correct?

(a)   1 and 2 only

(b)   2 and 3 only

(c)   1 and 3 only

(d)   1, 2 and 3
Answer(d)
  ExplanationStatement 1 is correct: Digital India Land Records Modernization Programme (erstwhile National Land Record Modernization Programme-) was revamped and converted as a Central Sector Scheme with effect from 1st April, 2016 with 100% funding by the Centre.

Statement 2 is correct: The scheme has the following components (i) Setting up of Modern Record Room (Teshil) (ii) Survey/re-survey (iii) Data entry/re-entry (iv) Digitization of cadastral Maps/FMBs/Tippans (v) State Level Data Centre (vi) Computerization of Registration process.

Statement 3 is correct: Currently, the Records of Rights in States and Union Territories are maintained in local languages. The linguistic barriers pose serious challenges for access of information and usage in understandable form. Hence, Records of Rights available in local language are being translated into any of the 22 Schedule VIII languages of the Constitution.
More Information
Reason/ Motivation of questionThe Department of Land Resources (Ministry of Rural Development) presented its year end review of 2023 where it highlighted the achievements of the DILRMP scheme.
Primary Source of Info.https://pib.gov.in/PressReleasePage.aspx?PRID=1989671
Nature of questionScheme (Agriculture)
 Difficulty levelMedium
 Qn. No. 49
SubjectEconomy & Social Development
 QuestionWith reference to the ‘Pradhan Mantri ‘Surakshit Matritva Abhiyan’, consider the following statements:

1.   This scheme guarantees a minimum package of antenatal care services to women in their second and third trimesters of pregnancy and six months post-delivery health care service in any government health facility

2.   Under this scheme, private sector health care providers of certain specialities can volunteer to provide services at nearby government health facilities.

Which of the statements given above is/are correct?

(a)   1 only

(b)  2 only

(c)   Both 1 and 2

(d)  Neither 1 nor 2
Answer(b)
 ExplanationThe Pradhan Mantri Surakshit Matritva Abhiyan (PMSMA) has been launched by the Ministry of Health & Family Welfare (MoHFW), Government of India. The program aims to provide assured, comprehensive and quality antenatal care, free of cost, universally to all pregnant women on the 9th of every month.

Statement 1 is incorrect: PMSMA guarantees a minimum package of antenatal care services to women in their 2nd / 3rd trimesters of pregnancy at designated government health facilities.

However, it does not provide any post-delivery health care service.

Statement 2 is correct: The programme follows a systematic approach for engagement with the private sector which includes motivating private practitioners to volunteer for the campaign developing strategies for generating awareness and appealing to the private sector to participate in the Abhiyan at government health facilities.
More Information
Reason/ Motivation of questionImportant scheme for the benefit of women.
 Primary Source of Info.https://pmsma.mohfw.gov.in/about-scheme/#about
Nature of questionScheme (Inclusive Growth)
 Difficulty levelMedium
Qn. No. 50
SubjectEconomy
 QuestionWith reference to the Pradhan Mantri Shram Yogi Maan-dhan (PM-SYM) Yojana, consider the following statements:

1.   The entry age group for enrolment in the scheme is 21 to 40 years.
2.   Age specific contribution shall be made by the beneficiary.
3.   Each subscriber under the scheme shall receive a minimum pension of 3,000 per month after attaining the age of 60 years.
4.   Family pension is applicable to the spouse and unmarried daughters.

Which of the statements given above is/are correct?

(a)   1, 3 and 4
(b)  2 and 3 only
(c)   2 only
(d)   1, 2 and 4
Answer(b)
ExplanationPradhan Mantri Shram Yogi Maan-dhan (PM-SYM) is a pension scheme meant for old age protection and social security of Unorganised Workers (UW). There are estimated 42 crore such unorganised workers in the country. Features: It is a voluntary and contributory pension scheme, under which thesubscriber would receive a minimum assured pension of Rs 3000/- per month after attaining the age of 60 years (Statement 3 is correct) and if the subscriber dies, the spouse of the beneficiary shall be entitled to receive 50% of the pension as family pension.

Family pension is applicable only to spouses. (Statement 4 is incorrect) Eligibility Criteria: Should be an unorganised worker (UW)Entry age between 18 and 40 years (Statement 1 is incorrect)

Monthly Income Rs 15000 or below Contribution by the UW Subscriber: Age-specific, through ‘auto-debit’ facility from his/ her savings bank account/ Jan- Dhan account from the date of joining PM-SYM till the age of 60 years as per the chart given in the scheme. The Central Government will also give equal matching contribution in his pension account. (Statement 2 is correct)
 More Information
Reason/ Motivation of questionScheme in line with the government’s objective of promoting financial inclusion.
 Primary Source of Info.https://labour.gov.in/brief-pm-sym
Nature of questionScheme (Inclusive Growth)
Difficulty levelMedium
Qn. No. 51
SubjectGeography
 Question Consider the following statements:

Statement-I: The atmosphere is heated more by incoming solar radiation than by terrestrial radiation Statement-II: Carbon dioxide and other greenhouse gases in the atmosphere are good absorbers of long wave radiation. Which one of the following is correct in respect of the above statements?

(a)    Both Statement-I and Statement-II are correct and Statement-II explains Statement-I

(b)    Both Statement-I and Statement-II are correct, but Statement-II does not explain Statement-I

(c)    Statement-I is correct, but Statement-II is incorrect

(d)    Statement-I is incorrect, but Statement-II is correct
Answer(d)
ExplanationMajor atmosphere is heated by outgoing longwave terrestrial radiation. CO2 etc are the GHGs and they absorb long wave radiation.
More Information
Reason/Motivation of questionBasic concept Statement I is incorrect: After being heated, the Earth itself becomes a radiating body and emits energy to the atmosphere in the form of long waves. This energy heats the atmosphere from below, a process known as terrestrial radiation. Statement II is correct: Long wave radiation is absorbed by atmospheric gases, particularly carbon dioxide and other greenhouse gases.
Primary Source of Info.NCERT, L-1 QIP
Nature of questionBasic
Difficulty levelEasy
Qn. No. 52
SubjectGeography
QuestionConsider the following statements:

Statement-I: Thickness of the troposphere at the equator is much greater as compared to poles.

Statement-II: At the equator, heat is transported to great heights by strong convectional currents. Which one of the following is correct in respect of the above statements?

(a)    Both Statement-I and Statement-II are correct and Statement-II explains Statement-I

(b)    Both Statement-I and Statement-II are correct, but Statement-II does not explain Statement-I

(c)    Statement-I is correct, but Statement-II is incorrect

(d)    Statement-I is incorrect, but Statement-II is correct
Answer(a)
ExplanationStatement 1 is factually correct. Tropospheric height is dependent upon temperature and convection currents at equator performs heat transfer to atmosphere effectively.
More Information
Reason/ Motivation of question Basic concepts
Primary Source of Info.NCERT, L-1 QIP, GS Foundation Program Statement I is factually correct: Tropospheric height is dependent upon temperature and convection currents at equator performs heat transfer to the atmosphere effectively.

Statement II is correct: The troposphere is thickest at the equator because strong convectional currents transport heat to great heights.
Nature of questionBasic
Difficulty levelEasy
Qn. No. 53
Subject Geography
QuestionConsider the following:

1. Pyroclastic debris
2. Ash and dust
3. Nitrogen compounds
4. Sulphur compounds

How many of the above are products of volcanic eruptions?

(a)  Only one
(b) Only two
(c)  Only three
(d)  All four
Answer (d)
ExplanationVolcanic eruptions involve emissions of many gasses including oxides of Nitrogen and Sulphur, pyroclastic material, ash and dust.
More Information
Reason/ Motivation of question Basic concept
Primary Source of Info.NCERT
Nature of questionBasic
Difficulty levelEasy
Qn. No. 54
Subject Geography
QuestionWhich of the following is/are correct inference/inferences from isothermal maps in the month of January?

1. The isotherms deviate to the north over the ocean and to the south over the continent.

2. The presence of cold ocean currents, Gulf Stream and North Atlantic Drift make the North Atlantic Ocean colder and the isotherms bend towards the north.

Select the answer using the code given below

(a)    1 only
(b)    2 only
(c)    Both 1 and 2
(d)    Neither 1 nor 2
Answer (a)
ExplanationSpecific heat of water is more compared to land and presence of gulf stream increase the temperature of North Atlantic ocean and produce positive thermal anomaly thereby bending the isotherms towards north in Oceans.
More Information
Reason/ Motivation of question Basic conceptual
Primary Source of Info.NCERT, L-1 QIP
Nature of questionBasic
Difficulty levelModerate
Qn. No. 55
Subject Geography
QuestionWhich of the following countries are well known as the two largest cocoa producers in the world?

(a)    Algeria and Morocco

(b)    Botswana and Namibia

(c)    Côte d’Ivoire and Ghana

(d)    Madagascar and Mozambique
Answer (c)
ExplanationFactual. The Ivory Coast and Ghana are by far the two largest producers of cocoa, accounting for more than 50 percent of the world´s cocoa
More Information
Reason/ Motivation of question Current affairs
Primary Source of Info.Frontline magazine, DNS
Nature of questionCurrent affairs
Difficulty levelDifficult
Qn. No. 56
Subject Geography
QuestionWith reference to the Himalayan rivers joining the Ganga downstream of Prayagraj from West to East, which one of the following sequences is correct?

(a)   Ghaghara-Gomati – Gandak – Kosi

(b)   Gomati Ghaghara Gandak Kosi

(c)   Ghaghara Gomati Kosi Gandak

(d)   Gomati Ghaghara Kosi Gandak
Answer (b)
ExplanationThe Gomti River merges with the Ganga near Varanasi, while the Ghaghara River joins the Ganga near Chhapra. The Gandak River converges with the Ganga at Hajipur, and the Kosi River joins the Ganga near the Katihar district of Bihar.
More Information
Reason/ Motivation of question Important rivers
Primary Source of Info.Oxford Atlas, L-4 QIP
Nature of questionBasic
Difficulty levelDifficult
Qn. No. 57
Subject Geography
QuestionConsider the following statements:

Statement-I: Rainfall is one of the reasons for weathering of rocks.

Statement-II: Rain water contains carbon dioxide in solution.

Statement-III: Rain water contains atmospheric oxygen. Which one of the following is correct in respect of the above statements?

(a)   Both Statement-II and Statement-III are correct and both of them explain Statement-I

(b)   Both Statement-II and Statement-III are correct, but only one of them explains Statement-I

(c)   Only one of the Statements II and III is correct and that explains Statement-I

(d)   Neither Statement-II nor Statement-III is correct
Answer (a)
ExplanationNormal Rain has the presence of mild carbonic acid and Acid rain has the presence of Nitric and sulphuric acid as well. E.g. of weathering by rainfall is dissolution of limestone and chalk rock in Serbia and Britain respectively thereby developing Karst Topography. Rainwater contains oxygen in it received through contact with Atmosphere.
More Information
Reason/ Motivation of question Basic concept
Primary Source of Info.NCERT
Nature of questionBasic
Difficulty levelModerate
Qn. No. 58
Subject Geography
QuestionConsider the following countries:

1.    Finland

2.    Germany

3.   Norway

4.    Russia

How many of the above countries have a border with the North Sea?

(a)   Only one (b)   Only two (c)   Only three (d)   All four
Answer (b)
ExplanationFactual map based Map of countries surrounding North Sea.
More Information
Reason/Motivation of question Important map location
Primary Source of Info.Oxford Atlas, L-3 QIP
Nature of questionBasic
Difficulty levelEasy
Qn. No. 59
SubjectGeography
QuestionConsider the following information:  

Waterfall Region River
1. Dhauandhar Malwa Narmada
2. Hundru Chota Nagpur Subarnarekha
3. Gersoppa Western Ghats Netravati

In how many of the above rows is the given information correctly matched?

(a)    Only one (b)    Only two (c)    All three (d)    None
Answer(a)
ExplanationFactual The Dhuandhar Falls is located on the Narmada River lies in region of Baghelkhand. The Hundru Falls at Ranchi is situated at a distance of 45 kilometers from the main city of Ranchi. The Hundru Falls Ranchi is created on the course of the Subarnarekha River, where is falls from a height of 320 feet creating the highest water falls of the state. Jog Falls, a major attraction in Karnataka, is located in the Shimoga district of Karnataka. Also known by alternative names of Gerusoppe Falls, Gersoppa Falls, and Jogada Gundi, it is one of the highest waterfalls in India.  Jog Falls is a waterfall on the Sharavati river located in Karnataka, India.
More Information
Reason/ Motivation of question Important waterfalls of India
Primary Source of Info.NCERT,  L4 QIP
Nature of questionBasic
Difficulty levelModerate
Qn. No. 60
Subject Geography
QuestionConsider the following information:  

Region Name of the mountain Type of Mountain
1. Central Asia Vosges Fold mountain
2. Europe Alps Block mountain
3. North America Appalachians Fold mountain
4. South America Andes Fold mountain

In how many of the above rows is the given information correctly matched?

(a)    Only one (b)   Only two (c)    Only three (d)    All four
Answer (b)
ExplanationFactual The Vosges are a block mountain in Eastern France, near its border with Germany. The Alps fold mountain range is located in Central Europe and covers Austria, Italy, Switzerland, Germany and France. Appalachians is Fold mountain located in North America and Andes is fold mountain and located in south America.
More Information
Reason/ Motivation of question Important mountains
Primary Source of Info.NCERT
Nature of questionBasic
Difficulty levelEasy
Qn. No. 61
SubjectEnvironment
QuestionThe organisms “Cicada, Froghopper and Pond skater” are:

(a)   Birds (b)   Fish (c)   Insects (d)   Reptiles
Answer(c)
ExplanationCicada- (family Cicadidae), family of more than 3,000 species of sound-producing insects. Cicadas are found in tropical and temperate areas worldwide and occur in deserts, grasslands, and forests. Cicadas have long been used in folk medicines, as religious and monetary symbols, and as an important source of food for humans and many other organisms.The froghoppers, or the superfamily Cercopoidea, are a group of hemipteran insects in the suborder Auchenorrhyncha. Adults are capable of jumping many times their height and length, giving the group their common name, but many species are best known for their plant-sucking nymphs which produce foam shelters, and are referred to as “spittlebugs”.The Gerridae are a family of insects in the order Hemiptera, commonly known as water striders, water skeeters, water scooters, water bugs, pond skaters, water skippers, water gliders, water skimmers or puddle flies.
More Information
Reason/ Motivation of question Current Affairs & Important recurring theme of UPSC: Species Biodiversity
Primary Source of Info.Rau’s IAS Foundation Magazine (Environment) Rau’s IAS DNS – 09/04/2024 – https://www.youtube.com/watch?v=E_lItnNc6A8&ab_channel=Rau%27sIAS Study Circle%28Since1953%29

https://www.thehindu.com/news/national/kerala/commonly-found-cicada-species-sheds-its-foreign-tag-to-embrace-an-indian-identity/article67087350.ece
Nature of questionFactual
Difficulty levelModerate
Qn. No.62
SubjectEnvironment
QuestionConsider the following statements:

Statement-I: Many chewing gums found in the market are considered pollution. a source of environmental
Statement-II: Many chewing gums contain plastic as gum base. Which one of the following is correct in respect of the above statements?

(a)   Both Statement-I and Statement-II are correct and Statement-II explains Statement-I

(b)   Both Statement-I and Statement-II are correct, but Statement-II does not explain Statement-I

(c)   Statement-I is correct, but Statement-II is incorrect

(d)   Statement-I is incorrect, but Statement-II is correct
Answer(a)
ExplanationStatement 1 is correct – Modern chewing gum is made of non-biodegradable hydrophobic polymers together with artificial sweeteners and flavours.

So, chewing this sort of synthetic material over a long time could produce some adverse effects. After chewing, most individuals throw the waste part of chewing gum everywhere, resulting in environmental trash known as ‘gum pollution. Each year, chewing gum generates more than 105 tonnes of “plastic” garbage. Thus, the discarded non-biodegradable residue of the gum produces plastic pollution.

Statement 2 is also correct – A key ingredient used in making chewing gum – “gum base” – actually contains polyvinyl acetate, a plastic which does not biodegrade once the gum is disposed of. Both Statement-I and Statement-II are correct and Statement-II explains Statement-I
 More Information
Reason/ Motivation of question Environmental Plastic Pollution & Important recurring theme of UPSC
Primary Source of Info.https://www.researchgate.net/publication/357845480_Improper_Disposal_of_Non-biodegradable_Chewing_Gum_is_One_of_the_Biggest_Threats_to_Our_ Ecology_A_Review#:~:text=After%20chewing%2C%20most%20indivi duals%20throw,the%20gum%20produces%20plastic%20pollution.
Nature of questionConceptual
Difficulty levelEasy
Qn. No. 63
SubjectEnvironment
QuestionConsider the following pairs:  

Country Animal found in its natural habitat
1. Brazil Indri
2. Indonesia Elk
3. Madagascar Bonobo

How many of the pairs given above are correctly matched?

(a)   Only one (b)   Only two (c)   All three (d)   None
Answer(d)
ExplanationIndri – It is a slender, long-limbed primate found in the forests of Madagascar. The largest of the lemurs, it is 60–70 cm (24–28 inches) long, with a rudimentary tail and large hands and feet. The round head has a pointed face and round, furry ears. Active during the day and thoroughly arboreal, the indri clings to trees and climbs in an upright position as it feeds on leaves, fruit, flowers, and other vegetation. The indri is an endangered species found only in remote parts of northeastern Madagascar, where it is protected by law.

Elk – Elk, the largest and most advanced subspecies of red deer, found in North America and in high mountains of Central Asia. It is a member of the deer family. They evolved as fast endurance runners that are very difficult to catch even with the best of horses, particularly in broken terrain. Bonobos and chimpanzees look very similar and both share 98.7% of their DNA with humans—making the two species our closest living relatives. Bonobos are usually a bit smaller, leaner, and darker than chimpanzees.

Wild bonobos can only be found in forests south of the Congo River in the Democratic Republic of Congo (DRC). They weren’t recognized as a separate species until 1929.
 More Information
Reason/Motivation of question Important recurring theme of UPSC: Species Biodiversity

https://indianexpress.com/article/opinion/editorials/bonobos-are-not-so-nice-just-like-humans-9272299/
Primary Source of Info.https://india.mongabay.com/2023/06/facing-space-and-resource-crunch-north-east-states-to-build-new-shelters-for-seized-wildlife/  

https://www.nationalgeographic.com/animals/article/160325-wolf-pack-kills-19-wolves-surplus-killing-wyoming  

https://indianexpress.com/article/opinion/editorials/bonobos-are-not-so-nice-just-like-humans-9272299/
Nature of questionFactual
Difficulty levelHard
Qn. No. 64
SubjectEnvironment
QuestionConsider the following statements regarding World Toilet Organization:

1.    It is one of the agencies of the United Nations.
2.    World Toilet Summit, World Toilet Day and World Toilet College are initiatives of this organization, the to inspire action to tackle the global sanitation crisis.
3.    The main focus of its function is to grant funds to the least developed countries and developing countries to achieve the end of open defecation.

Which of the statements given above is/are correct?

(a)   2 only (b)   3 only (c)   1 and 2 (d)   2 and 3
Answer(a)
ExplanationStatement 1 is incorrect: World Toilet Organization (WTO) is a global non-profit committed, founded in 2001, to improving toilet and sanitation conditions worldwide.

Statement 2 is correct: WTO established World Toilet Day and the World Toilet Summit in 2001; this was followed by the World Toilet College in 2005. Through its various initiatives, WTO continues to provide an international platform for toilet associations, governments, academic institutions, foundations, UN agencies and corporate stakeholders to exchange knowledge and leverage media and corporate support in an effort to promote clean sanitation and public health policies.

Statement 3 is incorrect: World Toilet Organization is one of the few organizations whose sole focus is on toilets and sanitation. Breaking the silence on the sanitation crisis is at the heart of what they do.WTO empowers individuals through education, training and building local marketplace opportunities to advocate for clean and safe sanitation facilities in their communities.
 More Information
Reason/Motivation of question   Environmental Pollution  
Primary Source of Info.https://worldtoilet.org/web-agency-gb-about-us/
Nature of questionFactual
Difficulty levelHard
Qn. No. 65
SubjectEnvironment
QuestionConsider the following statements:

1.    Lions do not have a particular breeding season.
2.    Unlike most other big cats, cheetahs do not roar.
3.    Unlike male lions, male leopards do not proclaim their territory by scent marking.

Which of the statements given above are correct ?

(a)   1 and 2 only (b)   2 and 3 only (c)   1 and 3 only (d)   1,2 and 3
Answer(a)
ExplanationStatement 1 is correct : Most higher mammals do not have any fixed breeding season. Similarly, Lions can breed throughout the year, and there is no specific breeding season for them.Statement 2 is correct: Cheetahs cannot roar; instead, they make a variety of other vocalizations, such as purring, chirping, and growling.Statement 1 is incorrect: Male leopards, like many other big cats, do use scent marking to proclaim their territory.
 More Information
Reason/Motivation of question Important recurring theme of UPSC: Species Biodiversity
Primary Source of Info.Rau’s IAS foundation Magazine – (Environment) & answer can be derived through class notes.
Nature of questionConceptual
Difficulty levelEasy
Qn. No.66
SubjectEnvironment
QuestionWhich one of the following is the correct description of “100 Million Farmers”?

(a)   It is a platform for accelerating the transition towards food and water systems that are net-zero (carbon), nature-positive and that aims to increase farmer resilience.

(b)   It is an international alliance and a network of individuals and farming organisations interested in supporting and strengthening the development of the organic animal husbandry.

(c)   It is a digital platform fully integrated with service providers and built on blockchain that lets buyers, sellers and third parties trade fertilizers quickly and securely.

(d)   It is a platform with the mission of encouraging the farmers to form Farmer Product Organisations or Agribusiness Consortiums, thus facilitating the access to global open markets to sell their products.
Answer(a)
Explanation100 Million Farmers is a platform, initiated by the World Economic Forum, supporting private and public leaders to position food and farmers as central pillars on the global climate and nature agenda, and accelerate collective action to scale climate-and nature-friendly agricultural practices and water systems by 2030
 More Information
Reason/Motivation of question Climate Change & Sustainable Agriculture
Primary Source of Info.
Nature of questionFactual
Difficulty levelModerate
Qn. No. 67
Subject Science & Tech
QuestionConsider the following:

1.    Battery storage
2.    Biomass generators
3.    Fuel cells
4.    Rooftop solar photovoltaic units

How many of the above are considered “Distributed Energy Resources”?

(a)   Only one (b)   Only two (c)   Only three (d)   All four
Answer (d)
ExplanationAll the above options represent decentralised energy resources as in each of these methods the production and consumption can happen at the same place and does not require transmission from a central production centre.
More Information
Reason/Motivation of question Renewable energy is an important component of a low-carbon future for India. In addition, decentralised sources are, as they can reduce the cost of transmission and losses during transmission. This is why it is preferred for which even rooftop solar generation is being given impetus in India.
Primary Source of Info.Compass + Class notes
Nature of questionConceptual
Difficulty levelEasy
Qn. No. 68
SubjectEnvironment
QuestionWhich one of the following shows a unique relationship with an insect that has coevolved with it and that is the only insect that can pollinate this tree?

(a)    Fig
(b)    Mahua
(c)    Sandalwood
(d)    Silk cotton
Answer(a)
ExplanationThe fig tree and the fig wasp exhibit a unique relationship where the fig wasp is the only insect that can pollinate the fig tree. This relationship is a classic example of coevolution, where both species have evolved together to become mutually dependent. The fig wasp lays its eggs inside the fig fruit, and in the process, it pollinates the tree, allowing it to reproduce.
More Information
Reason/ Motivation of question Important recurring theme of UPSC: Species Biodiversity & Biotic Interaction
Primary Source of Info.Rau’s IAS Class Notes – Topic – Biotic Interaction (Oviposition) https://www.sciencedirect.com/science/article/abs/pii/ S0169534703000624
Nature of questionFactual
Difficulty levelModerate
Qn. No. 69
Subject  Environment
QuestionConsider the following:

1.   Butterflies
2.   Fish
3.   Frogs How many of the above have poisonous species among them?

(a)   Only one (b)   Only two (c)   All three (d)   None
Answer (c)
ExplanationButterflies – Some species of butterflies, like those in the family Nymphalidae, are known to be poisonous due to the toxins they accumulate from the plants they eat during their larval stage. Eg-monarch butterflyFish – There are numerous species of poisonous fish, such as blowfish, balloon fish, toads, sunfish, porcupine fish, toadfish, globefish, or swellfish which contain tetrodotoxin, a potent neurotoxin.FrogsMany frog species, particularly those from the Dendrobatidae family (commonly known as dart frogs), are famous for being poisonous. They derive their toxicity from the insects and other small animals they consume.
More Information
Reason/ Motivation of question Rau’s IAS Foundation Magazine – Environment Important recurring theme of UPSC: Species Biodiversity & Ecological Adaptations

https://www.thehindu.com/news/national/kerala/fishers-encounter-yet-another-trouble-in-pufferfish/article67361266.ece

https://www.thehindu.com/sci-tech/science/sci-five-the-hindu-science-quiz-on-frogs/article68001694.ece
Primary Source of Info.Rau’s IAS Class Notes- Topic-Ecological Adaptations
https://www.sciencedirect.com/topics/pharmacology-toxicology-and-pharmaceutical-science/toxic-fish
Nature of questionFactual
Difficulty levelEasy
Qn. No. 70
Subject  Environment
QuestionConsider the following:

1.    Cashew
2.    Papaya
3.    Red sanders

How many of the above trees are actually native to India?

(a)   Only one
(b)   Only two
(c)   All three
(d)   None
Answer (a)
ExplanationAmong the listed plants, only Red sanders (Pterocarpus santalinus) is native to India. Cashew (originally from Brazil) and Papaya (originally from the tropics of the Americas) were introduced to India and other parts of the world through exploration and trade.
More Information
Reason/ Motivation of question Important recurring theme of UPSC: Endemic Species Biodiversity  
https://timesofindia.indiatimes.com/india/india-removed-from-negative-list-for-export-of-red-sanders/articleshow/105193561.cms  

https://indianexpress.com/article/cities/mumbai/ed-attaches-red-sanders-smuggler-badshah-maliks-properties-9204884/
Primary Source of Info.Rau’s IAS Compass; Raus’s IAS Class Notes
Nature of questionFactual
Difficulty levelEasy
Qn. No. 71
SubjectEconomy
QuestionConsider the following airports:

1.    Donyi Polo Airport
2.    Kushinagar International Airport
3.    Vijayawada International Airport

In the recent past, which of the above have been constructed as Greenfield projects?

(a)   1 and 2 only
(b)   2 and 3 only
(c)   1 and 3 only
(d)   1, 2 and 3
Answer(a)
Explanation According to the annual report of the Ministry of Civil Aviation, Donyi Polo and Kushinagar airports are Greenfield airports.

However, Vijayawada Airport is an existing airport which has been upgraded. Hence, the right answer is 1 and 2 only.
More Information
 Reason/Motivation of question Construction of Greenfield Airports by Government
 Primary Source of Info.https://www.civilaviation.gov.in/sites/default/files/2023-07/Annual%20Report%20of%20MoCA%202022-2023%20English.pdf
Nature of questionCurrent Affairs (Infrastructure)
 Difficulty levelDifficult
Qn. No. 72
SubjectGeography
QuestionWith reference to “water vapour”, which of the following statements is/are correct?

1.   It is a gas, the amount of which decreases with altitude.

2.    Its percentage is maximum at the poles.

Select the answer using the code given below:

(a)   1 only
(b)   2 only
(c)   Both 1 and 2
(d)   Neither 1 nor 2
Answer(a)
Explanation Gravity force is highest at the earth surface because of which relatively high number of gaseous molecules are present close to the earth’s surface including water vapor and other green house gasses.
Evaporation is low at poles.
More Information
Reason/ Motivation of question Basic conceptual
Primary Source of Info.NCERT, L-1&2 QIP
Nature of questionBasic
Difficulty levelModerate
Qn. No. 73
SubjectGeography
QuestionConsider the following description:

1.    Annual and daily range of temperatures is low.
2.    Precipitation occurs throughout the year.
3.    Precipitation varies between 50 cm 250 cm.

What is this type of climate?

(a)   Equatorial climate
(b)   China type climate
(c)   Humid subtropical climate
(d)   Marine West Coast climate
Answer(d)
 Explanation Marine West Coast climate receives precipitation because of its proximity with marine areas such as Pacific waters and Tasman sea. Moderating influence of sea do not allow much variation in the temperature of this climatic zone.
More Information
 Reason/Motivation of question Basic conceptual
 Primary Source of Info.NCERT, L-6 QIP
Nature of questionBasic
 Difficulty levelModerate
Qn. No. 74
SubjectGeography
QuestionWith reference to “Coriolis force”, which of the following statements is/are correct?

1.    It increases with increase in wind velocity.

2.    It is maximum at the poles and is absent at the equator

Select the answer using the code given below:

(a)   1 only
(b)   2 only
(c)   Both 1 and 2
(d)   Neither 1 nor 2
Answer(c)
 Explanation Fc=2wVSinq Fc: Coriolis force per unit mass V: Velocity of the object q Latitude w Rotational velocity of earth (it is constant)
More Information
Reason/ Motivation of question Basic conceptual
 Primary Source of Info.NCERT, L-1 QIP
Nature of questionBasic
 Difficulty levelEasy
Qn. No. 75
SubjectGeography
Question On June 21 every year, which of the following latitude(s) experience(s) a sunlight of more than 12 hours?
1.    Equator
2.    Tropic of Cancer
3.   Tropic of Capricorn
4.   Arctic Circle

Select the correct answer using the code given below:

(a)   1 only
(b)   2 only
(c)   3 and 4
(d)   2 and 4
Answer(d)
ExplanationMaximum insolation on 21st June is in Northern Hemisphere (Summer solstice of Northern hemisphere) ie To the North of Tropic of Cancer, high duration for insolation.
 More Information
Reason/ Motivation of question Basic concept
Primary Source of Info.NCERT, L1 QIP
Nature of questionBasic
Difficulty levelEasy
Qn. No. 76
SubjectEnvironment
QuestionOne of the following regions has the world’s largest tropical peatland, which holds about three years worth of global carbon emissions from fossil fuels; and the possible destruction of which can exert detrimental effect on the global climate.

Which one of the following denotes that region?

(a) Amazon Basin (b) Congo Basin (c) Kikori Basin (d) Rio de la Plata Basin
Answer(b)
ExplanationThe Congo Basin is home to the world’s largest tropical peatlands, along with Brazil and Indonesia. The peat swamp forest of the Congo Basin stores around 29 billion tons of carbonapproximately equivalent to three years’ worth of global greenhouse gas emissions – while the Basin as a whole absorbs nearly 1.5 billion tons of carbon dioxide a year . The Basin stretches across six countries- Cameroon, Central African Republic, Democratic Republic of the Congo, Congo, Equatorial Guinea and Gabon.
More Information
Reason/ Motivation of question Climate Change & Destruction of Ecosystem
Primary Source of Info.https://www.iucn.org/news/climate-change/201803/ministers-commit- protecting-world%E2%80%99s-largest-tropical-peatland

https://www.unep.org/news-and-stories/story/critical-ecosystems-congo-basin-peatlands#:~:text=The%20Congo%20Basin%20is%20home,along%20 with%20Brazil%20and%20Indonesia.
Nature of questionFactual
Difficulty levelEasy
Qn. No. 77
SubjectEnvironment
Question With reference to perfluoroalkyl and polyfluoroalkyl substances (PFAS) that are used in making many consumer products, consider the following statements:

1.     PFAS are found to be widespread in drinking water, food and food packaging material.
2.     PFAS are not easily degraded in the environment.
3.     Persistent exposure to PFAS can lead to lead to bioaccumulation in animal bodies.

Which of the statements given above are correct?

(a)    1 and 2 only
(b)    2 and 3 only
(c)    1 and 3 only
(d)    1, 2 and 3
Answer(d)
ExplanationStatement 1 is correct: Perfluoroalkyl and polyfluoroalkyl substances (PFAS) are found widespread in drinking water, foods, food packaging materials and other consumer products. They have exceptional resistance to water or heat.

Statement 2 is correct: PFAS molecules have a chain of linked carbon and fluorine atoms. Because the carbon-fluorine bond is one of the strongest, these chemicals do not degrade easily in the environment.They are persistent compounds often referred to as forever chemicals which do not degrade easily.

Statement 3 is correct: People may encounter different PFAS chemicals in various ways. Over time, people may take in more of the chemicals than they excrete, a process that leads to bioaccumulation in bodies.
More Information
Reason/ Motivation of question Plastic Pollution & Bioaccumulation https://economictimes.indiatimes.com/magazines/panache/from-polar-bears-to-groundwater-nature-is-riddled-with-forever-chemicals/articleshow/109045121.cms?from=mdr
Primary Source of Info.https://www.niehs.nih.gov/health/topics/agents/pfc#:~:text= PFAS%20remain%20in%20the%20environment,leads%20to%20 bioaccumulation%20in%20bodies. https://pubmed.ncbi.nlm.nih.gov/32476019/
Nature of questionConceptual
Difficulty levelDifficult
Qn. No. 78
SubjectEnvironment
QuestionConsider the following:

1. Carabid beetles
2. Centipedes
3. Flies
4. Termites
5. Wasps

Parasitoid species are found in how many of the above kind of organisms?

(a) Only two (b) Only three (c) Only four (d) All five
Answer(d)
ExplanationParasitoid species, which are organisms that live on or within a host and ultimately kill it, are found among several different types of organisms listed above. Carabid beetles – Some species of these beetles have been found to be parasitic, particularly in their larval stages, on other insects.Centipedes – Centipedes can host a variety of parasites, including mites, nematodes, fungi, and protozoa. These parasites can affect the health, behavior, and survival of centipedesFlies – Many species of flies, particularly within the families Tachinidae and some other groups, are known to be parasitoids.Termites – Some termite species host numerous parasitic arthropod species, called termitophiles, others host none.Wasps – Many wasps, especially within the Hymenoptera order, are famous for being parasitoids, with numerous species laying their eggs in or on other insects. Based on this information, parasitoid species are found in three of the above groups — Carabid beetles, flies, and wasps. Thus, the answer is: (b) Only three
More Information
Reason/ Motivation of question Important recurring theme of UPSC: Species Biodiversity & Biotic Interaction
Primary Source of Info.https://www.sciencedirect.com/topics/immunology-and-microbiology/carabidae

https://www.sciencedirect.com/topics/biochemistry-genetics-and-molecular-biology/parasitoid

https://www.sciencedirect.com/topics/agricultural-and-biological-sciences/tachinidae

https://www.ncbi.nlm.nih.gov/pmc/articles/PMC9311137/ https://ipm.ucanr.edu/natural-enemies/tachinid-flies/#gsc.tab=0
Nature of questionFactual
Difficulty levelHard
Qn. No. 79
SubjectMiscellaneous
QuestionConsider the following plants:

1.  Groundnut

2. Horse-gram

3. Soybean How many of the above belong to the pea family?

(a)  Only one
(b) Only two
(c) All three
(d) None
Answer(c)
ExplanationThe Pea Family is known by the scientific name Fabaceae. All the above three crops Horsegram, Groundnut and Soybean belong to the Fabacae family and hence belong to the Pea Family.
More Information
Reason/ Motivation of question Pulses are important crops of India.
Primary Source of Info. 
Nature of questionFactual
Difficulty levelDifficult
Qn. No. 80
SubjectEnvironment
Question Consider the following statements:

Statement-I: The Indian Flying Fox is placed under the “vermin” category in (Protection) Act, 1972. the Wild Life

Statement-II: The Indian Flying Fox feeds on the blood of other animals.

Which one of the following is correct in respect of the above statements?

(a) Both Statement-I and Statement-II are correct and Statement-II explains Statement-I

(b) Both Statement-I and Statement-II are correct, but Statement-II does not explain Statement-I

(c) Statement-I is correct, but Statement-II is incorrect

(d) Statement-I is incorrect, but Statement-II is correct
Answer(c)
ExplanationStatement 1 is correct: The nectar and fruit-eating flying fox (Pteropus giganteus) is generally considered a vermin as they raid orchards. It had a similar official status under Schedule V of the Wildlife (Protection) Act of 1972. Statement 2 is incorrect: They are considered vermin due to its destructive tendencies towards fruit farms. Humans in general dislike them for their noise.
 More Information
Reason/ Motivation of question Important recurring theme of UPSC: Species Biodiversity & Environmental Legislation
Primary Source of Info.https://www.thehindu.com/sci-tech/science/flying-fox-bats-for-vigilance-while-roosting/article67184575.ece#:~:text=Most%20bats%20forage %20at%20night,such%20as%20heat%20and%20light.  

https://timesofindia.indiatimes.com/city/gurgaon/they-may-be-called-vermins-but-fruit-bats-are-vital-for-ecosystem/articleshow/81263203.cms
Nature of questionFactual
Difficulty levelModerate
Qn. No. 81
SubjectEconomic & Social Development 
QuestionThe total fertility rate in an economy is defined as:

(a) the number of children born per 1000 people in the population in a year.

(b) the number of children born to a couple in their lifetime in a given population.

(c) the birth rate minus death rate.

(d) the average number of live births a woman would have by the end of her child-bearing age.
Answer(d)
ExplanationThe total fertility rate refers to the total number of live births that a hypothetical woman would have if she lived through the reproductive age group and had the average number of babies in each segment of this age group as determined by the age-specific fertility rates for that area. Another way of expressing this is that the total fertility rate is the ‘the average number of births to a cohort of women up to the end of the reproductive age period (estimated on the basis of the age-specific rates observed during a given period)’
More Information
Reason/ Motivation of question  
Primary Source of Info.https://ncert.nic.in/ncerts/l/lesy102.pdf
Nature of questionCore Concept (Population)
Difficulty levelEasy
Qn. No. 82
Subject Economy
QuestionConsider the following statements:

1. In India, Non-Banking Financial Companies can access the Liquidity Adjustment Facility window of the Reserve Bank of India.
2.  In India, Foreign Institutional Investors can hold the Government Securities (G-Secs).
3.  In India, Stock Exchanges can offer separate trading platforms for debts.

Which of the statements given above is/are correct?

(a) 1 and 2 only
(b) 3 only
(c) 1, 2 and 3
(d) 2 and 3 only
Answer(d)
ExplanationStatement 1 is incorrect: Liquidity Adjustment Facility (LAF) is a facility extended by RBI to the scheduled commercial banks (excluding Regional Rural Banks) and Primary Dealers to avail of liquidity in case of requirement or park excess funds with RBI in case of excess liquidity on an overnight basis against the collateral of G-Secs including SDLs. Basically, LAF enables liquidity management on a day to day basis. The operations of LAF are conducted by way of repurchase agreements (repos and reverse repos). Hence, non-banking financial companies cannot access the LAF window.

Statement 2 is correct: Foreign Institutional Investors (FIIs) are allowed to participate in the G-Secs market within the quantitative limits prescribed from time to time. Statement 3 is correct: The National Stock Exchange (NSE) launched the country’s first dedicated debt trading platform in May 2023. The separate debt trading platform will provide an opportunity to retail investors to invest in corporate bonds on a liquid and transparent exchange platform. It will also help institutions which are holders of corporate bonds an ideal platform to buy and sell at optimum prices and help corporates to get adequate demand, when they are issuing the bonds, according to a release here.
More Information
Reason/ Motivation of question In May 2023, the National Stock Exchange (NSE) launched the country’s first dedicated debt trading platform
Primary Source of Info.https://www.rbi.org.in/scripts/FS_FAQs.aspx?Id=79&fn=2757#6

https://www.rbi.org.in/commonperson/English/Scripts/FAQs.aspx?Id=711#9

https://www.business-standard.com/article/markets/nse-launches-separate-debt-trading-platform-113051300554_1.html
Nature of questionBanking & Finance; Capital Markets
Difficulty levelEasy
Qn. No. 83
SubjectEconomy
QuestionIn India, which of the following can trade in Corporate Bonds and Government Securities?

1. Insurance Companies

2. Pension Funds

3. Retail Investors

Select the correct answer using the code given below:

(a) 1 and 2 only
(b) 2 and 3 only
(c) 1 and 3 only
(d) 1, 2 and 3
Answer(d)
ExplanationParticipants in the Corporate Bonds and Government Securities Market include:

-Commercial banks
-Co-operative banks
-Regional rural banks
-Primary Dealers
Insurance companies.
-Mutual fundsProvident Funds
Pension funds.
-Foreign Portfolio Investors (FPIs)
-Corporates

Under the Retail Direct Scheme, RBI has allowed retail investors to directly trade in G-Secs in both primary and secondary markets.
More Information
Reason/ Motivation of question The Reserve Bank of India (RBI) has announced the launch of a mobile app for the Retail Direct portal. The mobile app will allow investors to invest in different types of government securities such as central government bonds, state government bonds, Treasury bills etc.
Primary Source of Info.https://www.rbi.org.in/commonperson/English/Scripts/FAQs.aspx?Id=711

https://www.rbi.org.in/commonman/English/Scripts/PressReleases.aspx?Id=3339
Nature of questionCapital Markets
Difficulty levelEasy
Qn. No. 84
SubjectEconomy
Question Consider the following:

1.  Exchange-Traded Funds (ETF)
2. Motor vehicles
3. Currency swap

Which of the above is/are considered financial instruments?

(a)  1 only
(b) 2 and 3 only
(c) 1, 2 and 3
(d) 1 and 3 only
Answer(d)
ExplanationAn ETF, or exchange traded fund, is a marketable security that tracks an index, a commodity, bonds, or a basket of assets like an index fund.Currency swaps are important financial instruments used by banks, investors, and multinational corporations.

In a currency swap, the parties agree in advance whether or not they will exchange the principal amounts of the two currencies at the beginning of the transaction. Motor Vehicles are not a financial instrument.
More Information
Reason/ Motivation of question  
Primary Source of Info.https://www.amfiindia.com/investor-corner/knowledge-center/etf.html

https://www.investopedia.com/terms/c/currencyswap.asp
Nature of questionCurrent Affairs (Banking & Finance)
Difficulty levelMedium
 Qn. No. 85
SubjectEconomy
Question With reference to the sectors of the Indian economy, consider the following pairs:  

Economic activity Sector
1. Storage of agricultural Produce Secondary
2. Dairy farm Primary
3. Mineral exploration Tertiary
4. Weaving cloth Secondary

How many of the pairs given above are correctly matched?

(a) Only one
(b) Only two
(c) Only three
(d) All four
Answer(b)
ExplanationOnly pairs 2nd and 4th are correct.

Primary Sector: When we produce a good by exploiting natural resources, it is an activity of the primary sector. This is because it forms the base for all other products that we subsequently make. Since most of the natural products we get are from agriculture, dairy, fishing, forestry, this sector is also called agriculture and related sectors.

Similarly, minerals and ores are also natural products.

Secondary Sector: The secondary sector covers activities in which natural products are changed into other forms through ways of manufacturing that we associate with industrial activity. The product is not produced by nature but has to be made and therefore some process of manufacturing is essential. This could be in a factory, a workshop or at home.Since this sector gradually became associated with the different kinds of industries that came up, it is also called as industrial sector

Examples:Using cotton fiber from the plant, we spin yarn and weave cloth. Using sugarcane as a raw material, we make sugar or gur. We convert earth into bricks and use bricks to make houses and buildings.

Tertiary Sector: These are activities that help in the development of the primary and secondary sectors. These activities, by themselves, do not produce a good but they are an aid or a support for the production process. Since these activities generate services rather than goods, the tertiary sector is also called the service sector

Example: Transport, storage, communication, banking, trade are some examples of tertiary activities. Goods that are produced in the primary or secondary sector would need to be transported by trucks or trains and then sold in wholesale and retail shops. At times, it may be necessary to store these in godowns. We also may need to talk to others over telephone or send letters (communication) or borrow money from banks (banking) to help production and trade.
More Information
Reason/ Motivation of question  
Primary Source of Info.https://ncert.nic.in/textbook/pdf/jess202.pdf
Nature of questionCore Concepts
Difficulty levelEasy
Qn. No. 86
SubjectScience & Tech
QuestionConsider the following materials:

1.   Agricultural residues
2.   Corn grain
3.   Wastewater treatment sludge
4.   Wood mill waste

Which of the above can be used as feedstock for producing Sustainable Aviation Fuel?

(a)   1 and 2 only
(b)   3 and 4 only
(c)   1, 2, 3 and 4
(d)   1, 3 and 4 only
Answer(c)
 Explanation Sustainable Aviation Fuel is integral to the strategy adopted by the aviation industry to go net-zero by 2050. It is essentially a  biofuel, and biofuels can be produced by all the above feedstocks as all of them have organic content.
More Information
Reason/ Motivation of question Alternative fuels are the way in the transport sector including aviation. Further the global aviation sector has committed to become net zero by 2050. Although the long-term option is hydrogen and electrification, in the medium term the options include biofuels. In addition, given the lower volumetric energy density of hydrogen and high requirements for aircrafts, the best available option is biofuels.
 Primary Source of Info.Commonly seen in news
Nature of questionConceptual application based
Difficulty levelEasy
Qn. No. 87
SubjectEconomy
QuestionWith reference to physical capital in Indian economy, consider the following pairs:  

Items Category
1. Farmer’s plough Working capital
2. Computer Fixed Capital
3. Yearn used by the weaver Fixed Capital
4. Petrol Working capital

How many of the above pairs are correctly matched ?

(a) Only one
(b)  Only two
(c) Only three
(d)  All four
Answer(b)
ExplanationOnly pairs 2nd and 4th are correctly matched

Fixed capital refers to investment in long-term assets.

-Management of fixed capital involves allocation of firm’s capital to different projects or assets with long-term implications for the business.
-These decisions are called investment decisions or capital budgeting decisions and affect the growth, profitability and risk of the business in the long run.
-These long-term assets last for more than one year.
-Investment in these assets would also include expenditure on acquisition, expansion, modernisation and their replacement.
-These decisions include purchase of land, building, plant and machinery, launching a new product line or investing in advanced techniques of production.
-Major expenditures such as those on advertising campaigns or research and development programmes having long term implications for the firm are also examples of capital budgeting decisions.

Examples: Farmer’s plough, computer are examples of fixed capital.

Working Capital: Apart from the investment in fixed assets every business organisation needs to invest in current assets. This investment facilitates smooth day-to-day operations of the business.
Examples of current assets, in order of their liquidity, are as under:

-Cash in hand/Cash at Bank
-Finished goods inventory
-Work in progress
-Raw materials
-Prepaid expenses

Examples: Yarn used by weavers and petrol are raw materials used in day-to-day functioning.
More Information
Reason/ Motivation of question  
Primary Source of Info.https://ncert.nic.in/textbook/pdf/lebs201.pdf
Nature of questionCore Concept
Difficulty levelDifficult
Qn. No. 88
SubjectScience & Tech
QuestionWhich one of the following words/phrases is most appropriately used to denote “an interoperable network of 3D virtual worlds that can be accessed simultaneously by millions of users, who can exert property rights over virtual items”?

(a) Big data analytics
(b) Cryptography
(c)  Metaverse
(d) Virtual matrix
Answer(c)
 Explanation Metaverse is said to be the future of internet or embodied internet in which the users will get an immersive experience of the web. This is essentially a network of 3-D virtual worlds made possible by a suite of technologies including AR/VR, blockchain, NFTs, SDN, CDN, 5G etc.
More Information
Reason/ Motivation of question Metaverse is touted to be the future of the internet. Big techs like Meta, games like fortnite, cities like Istanbul have all made 3D virtual worlds in recent times. It is one of the frontier technologies that is touted to drive the web 3.0 and seen to change important areas including urban planning, real estate sector, 3-D classrooms in education, hazardous industries to name a few.
 Primary Source of Info.Commonly seen in news
Nature of questionConceptual
Difficulty levelEasy
Qn. No. 89
SubjectEconomy
QuestionWith reference to the rule/rules imposed by the Reserve Bank of India while treating foreign banks, consider the following statements:

1.  There is no minimum capital requirement for wholly owned banking subsidiaries in India.
2.   For wholly owned banking subsidiaries in India, at least 50% of the board members should be Indian nationals.

Which of the statements given above is/are correct?

(a) 1 only
(b) 2 only
(c) Both 1 and 2
(d)  Neither 1 nor 2
Answer(b)
ExplanationStatement 1 is incorrect: The wholly owned subsidiaries (WOS) shall be required to maintain a minimum capital adequacy ratio of 10% of the risk weighted assets or as may be prescribed from time to time on a continuous basis from the commencement of operations.

Statement 2 is correct: In order to ensure that the board of directors of the WOS of foreign banks set up in India acts in the best interest of the local institution, RBI may, in line with the best practices in other countries, mandate that not less than 50 percent of the directors should be Indian nationals resident in India.
More Information
Reason/ Motivation of question  
Primary Source of Info.https://m.rbi.org.in/Scripts/bs_viewcontent.aspx?Id=2313#:~:text= Foreign%20banks%20with%20branch%20presence,BC
Nature of questionBanking & Finance
Difficulty levelMedium
Qn. No. 90
SubjectEconomy
QuestionWith reference to Corporate Social Responsibility (CSR) rules in India, consider the following statements:
1. CSR rules specify that expenditures that benefit the company directly or its employees will not be considered as CSR activities.
2. CSR rules do not specify minimum spending on CSR activities.

Which of the statements given above is/are correct?

(a) 1 only
(b) 2 only
(c) Both 1 and 2
(d) Neither 1 nor 2
Answer(a)
ExplanationStatement 1 is correct: Rule 2(1)(d) of the Companies (CSR Policy) Rules, 2014 defines CSR and the following activities are specifically excluded from being considered as eligible CSR activity: Activities undertaken in pursuance of normal course of business of the company.Activities undertaken outside India, except for training of Indian sports personnel representing any State or Union Territory at national level or India at international levelContribution of any amount, directly or indirectly, to any political party under section 182 of the Act; Activities benefiting employees of the company as defined in section 2(k) of the Code on Wages, 2019Sponsorship activities for deriving marketing benefits for products/services; (vi) Activities for fulfilling statutory obligations under any law in force in India.

Statement 2 is incorrect: Section 135(5) of the Act prescribes minimum spending obligation for the company. It ensures that the company spends, in every financial year, at least 2% of the average net profits of the company made during the three immediately preceding financial years.
More Information
Reason/ Motivation of question The Centre is weighing a proposal to tweak corporate social responsibility (CSR) norms for widening their scope and coverage besides reviewing the penalty structure to discourage companies from non-compliance.
Primary Source of Info.https://www.mca.gov.in/Ministry/pdf/FAQ_CSR.pdf
Nature of questionCurrent Affairs (Industry)
Difficulty levelMedium
Qn. No. 91
SubjectScience & Tech
Question With reference to radioisotope thermoelectric generators (RTGs), consider the following statements:
1.   RTGs are miniature fission reactors.
2.   RTGs are used for powering the onboard systems of spacecrafts.
3.    RTGs can use Plutonium-238, which is by-product a of weapons development.

Which of the statements given above are correct?

(a)   1 and 2 only
(b)   2 and 3 only
(c)   1 and 3 only
(d)   1, 2 and 3
Answer(b)
ExplanationRTGs or Radioisotope Thermo-electric Generators use a radioactive material which when decays produces heat. This heat is in turn used by a generator to produce electricity. This is particularly useful in spacecrafts that need continuous energy to go to outer space or ‘deep space’ where the solar energy reaching is low. So, C these devices have 2 parts, one is a heat source and another thermocouples. It uses the temperature difference between the hot side and cold side to produce energy. This is called the seebeck effect .

E.g., New Horizon which went to Pluto uses this kind of device. Voyage which went past our solar system recently also used RTGs. Thus options 2 and 3 are correct. Since fission essentially involves break-up of the neutron and not just a minor change (transmutation) option 1 is incorrect.
More Information
Reason/Motivation of question Question of nuclear technology is a common phenomenon in the paper. This time the examiner has tested one’s knowledge on the basic concept of radioactivity and its application.
Primary Source of Info.Compass
Nature of questionConceptual and application
Difficulty levelEasy
Qn. No. 92
SubjectScience & Tech
Question Consider the following statements:

Statement-I: Giant stars live much longer than dwarf stars.
Statement-II: Compared to dwarf stars, giant stars have a greater rate of nuclear reactions.
Which one of the following is correct in respect of the above statements?

(a) Both Statement-I and Statement-II are correct and Statement-I Statement-II explains
(b) Both Statement-I and Statement-II are correct, but Statement-II does not explain Statement-I
(c) Statement-I is correct, but Statement-II is incorrect
(d) Statement-I is incorrect, but Statement-II is correct
Answer(d)
ExplanationOwing to higher mass, higher temperatures are reached in giant stars due to gravitational contraction. This not only triggers nuclear fusion but also at a higher rate at which fusion occurs which in turn increases the temperature due to energy release setting a cycle of higher reaction rates and higher temperature. Thus, statement 2 is correct.

Given the higher rate of reactions giant stars exhaust their fuel faster than dwarf ones reducing its age which is essentially the time duration of fusion reactions. Thus, statement 1 is incorrect.
More Information
Reason/ Motivation of question Concepts related to universe/stellar evolution.
Primary Source of Info.—-
Nature of questionConcepts in astronomy
Difficulty levelModerate
Qn. No. 93
SubjectScience & Tech
QuestionWhich one of the following is synthesised in human body that dilates blood vessels and increases blood flow?

(a) Nitric oxide
(b) Nitrous oxide
(c) Nitrogen dioxide
(d) Nitrogen pentoxide
Answer(a)
ExplanationNitric oxide (NO) is a gas that is synthesised by nearly every type of cell in the human body in most body tissues, including the endothelial cells within the blood vessels. Nitric oxide promotes the dilation of blood vessels. It works by relaxing the inner walls of blood vessels, particularly the large arteries and veins, thereby enabling them to dilate. This enables blood to flow smoothly.
 More Information
Reason/ Motivation of question Basics of Biology
Primary Source of Info.–—
Nature of questionRandom
Difficulty levelModerate
Qn. No. 94
SubjectScience & Tech.
QuestionConsider the following activities:
1.  Identification of narcotics on passengers at airports or in aircraft.
2.  Monitoring of precipitation
3.  Tracking the migration of animals

In how many of the above activities can the radars be used?

(a) Only one
(b) Only two
(c) All three
(d) None
Answer(b)
ExplanationRadars utilise various principles such as the reflection and scattering of electromagnetic waves to detect, locate, and identify objects or phenomena.

Radars can be used in:

Monitoring of precipitation: Weather radars are commonly used to detect and track precipitation, such as rainfall and snowfall, for weather forecasting and monitoring purposes.

Tracking the migration of animals: Radars can be used to detect and track the movement of animals, particularly birds and other large wildlife, to study their migration patterns and behaviours. However, Radars cannot be utilised for Identification of narcotics on passengers at airports or in aircraft. Instead, X-ray imaging systems at ports could serve the purpose.
 More Information
Reason/ Motivation of question Basic question on Remote Sensing
Primary Source of Info.Compass
Nature of questionApplication
Difficulty levelEasy
Qn. No. 95
SubjectScience & Tech.
QuestionConsider the following aircraft:
1. Rafael
2. MiG-29
3. Tejas MK-1

How many of the above are considered fifth generation fighter aircraft?

(a) Only one (b) Only two (c) All three (d) None
Answer(d)
ExplanationNone of the aircraft listed – Rafael, MiG-29, and Tejas MK-1  are fifth-generation fighter aircraft. Rafael is a 4.5 generation multirole fighter aircraft developed by Dassault Aviation of France. MiG-29 is a 4th generation jet fighter aircraft developed in the Soviet Union. Tejas MK-1 is an indigenous 4.5 generation multirole fighter aircraft developed by India.
More Information
Reason/ Motivation of question Developments in Defence Technology
Primary Source of Info.Newspapers
Nature of questionFactual
Difficulty levelEasy to moderate
Qn. No. 96
SubjectScience & Tech.
QuestionIn which of the following are hydrogels used ?

1. Controlled drug delivery in patients
2. Mobile air-conditioning systems
3. Preparation of industrial lubricants

Select the correct answer using the code given below:

(a) 1 only
(b) 1 and 2 only
(c) 2 and 3 only
(d) 1, 2 and 3
Answer(d)
ExplanationHydrogels are increasingly seen in bio-manufacturing like scaffolding for tissue engineering, safe and biodegradable drug delivery systems etc. given its safe and biodegradable nature.

Therefore option 1 is correct. Hydrogels can be used in dehumidifying air and thus to give a cooling effect. Hence, they can be used in mobile air-conditioning systems. Thus, option 2 is correct. Hydrogen based lubricants are used in medical applications and also they can be used in the preparation of industrial lubricants. Thus, option 3 is correct.
 More Information
Reason/ Motivation of question Hydrogels are increasingly seen in bio manufacturing like scaffolding for tissue engineering, safe and biodegradable drug delivery systems etc. given its safe and biodegradable nature.
Primary Source of Info.—-
Nature of questionApplication-based
Difficulty levelModerate
Qn. No. 97
SubjectScience & Tech.
QuestionWhich one of the following is the exhaust pipe emission from Fuel Cell Electric Vehicles, powered by hydrogen?

(a)   Hydrogen peroxide
(b)   Hydronium
(c)   Oxygen
(d)   Water vapour
Answer(d)
ExplanationThe exhaust pipe emission from Fuel Cell Electric Vehicles (FCEVs) powered by hydrogen is water vapour. In a fuel cell, hydrogen (fuel) reacts with oxygen (oxidiser) to produce electricity, heat, and water. The chemical reaction that takes place in the fuel cell is: 2H2 + O2 → 2H2O Hence, the only by product of the fuel cell reaction is water, which is typically expelled as water vapour through the vehicle’s exhaust.
 More Information
Reason/ Motivation of question The future of transport is said to be a combination of hydrogen and electrification. Further, given the low volumetric density of hydrogen, FCEVs are said to be a better alternative to use hydrogen than IC engines. Thus, there is a push for FCEVs in India which is reflected in the Union Minister of Transport travelling to parliament in recent years in Toyota Mirai which is hydrogen powered FCEV.
Primary Source of Info.Often seen in news
Nature of questionConceptual
Difficulty levelEasy
Qn. No. 98
SubjectScience & Tech.
QuestionRecently, the term “pumped-storage hydropower” is actually and appropriately discussed in the context of which one of the following?

(a)   Irrigation of terraced crop fields
(b)   Lift irrigation of cereal crops
(c)   Long duration energy storage
(d)  Rainwater harvesting system
Answer(c)
ExplanationPumped-storage hydropower is discussed in the context of long-duration energy storage. It is a type of energy storage system that uses electricity to pump water from a lower reservoir to a higher reservoir. During times of high electricity demand, the stored water is released through turbines to generate electricity.
More Information
Reason/Motivation of question For the success of renewable energy, revolution energy storage is an important component given the intermittent nature of renewable energy.  Thus energy storage systems will be tested
Primary Source of Info.Compass
Nature of questionTerms related to energy storage
Difficulty levelEasy
Qn. No. 99
Subject Science & Tech.
Question“Membrane Bioreactors” are often discussed in the context of: (a) Assisted reproductive technologies (b) Drug delivery nanotechnologies (c) Vaccine production technologies (d) Wastewater treatment technologies
Answer (d)
ExplanationMembrane Bioreactors (MBRs) are advanced systems commonly used in municipal and industrial wastewater treatment applications. They combine a biological treatment process (use microorganisms to break down organic matter and other contaminants) with membrane filtration, allowing for highly efficient removal of contaminants from wastewater.
More Information
Reason/ Motivation of question Development in Waste-management technologies 
Primary Source of Info. 
Nature of question Application
Difficulty levelEasy
Qn. No. 100
SubjectEconomy
QuestionWith reference to the Indian economy, “Collateral Borrowing and Lending Obligations” are the instruments of: (a) Bond market (b) Forex market (c)  Money market (d) Stock market
Answer(c)
ExplanationThe Collateralized Borrowing and Lending Obligation (CBLO) market is a money market segment operated by the Clearing Corporation of India Ltd (CCIL). In the CBLO market, financial entities can avail short term loans by providing prescribed securities as collateral. In terms of functioning and objectives, the CBLO market is almost similar to the call money market.The uniqueness of CBLO is that lenders and borrowers use collateral for their activities. For example, borrowers of funds have to provide collateral in the form of government securities and lenders will get it while giving loans. There is no such need for collateral under the call money market.
More Information
Reason/ Motivation of question  
Primary Source of Info.https://rbi.org.in/Scripts/BS_CircularIndexDisplay.aspx?Id=2388
Nature of questionBanking & Finance
Difficulty levelEasy

UPSC Prelims Answer Key 2024 for GS Paper 1 (Set A)

Here, download answer key for Set – A.

UPSC PRELIMS 2024 ANSWER KEY - SET A

UPSC Prelims Answer Key 2024 for GS Paper 1 (Set B)

Here, download answer key for Set – B.

UPSC PRELIMS 2024 ANSWER KEY - SET B

UPSC Prelims Answer Key 2024 for GS Paper 1 (Set C)

Here, download answer key for Set – C.

UPSC PRELIMS 2024 ANSWER KEY - SET C

UPSC Prelims Answer Key 2024 for GS Paper 1 (Set D)

Here, download answer key for Set – D.

UPSC PRELIMS 2024 ANSWER KEY - SET D

Calculate your Prelims Score from Answer Key:

This answer key is prepared by the faculty team from Rau’s IAS. The official key is yet to be released this year, but you should be ready for the mains. Without wasting any time, you can check your score using the detailed and explained answer key provided above.

No. Questions100
Total Marks200
Marks for Correct Answer+2
Negative Marks for Incorrect Answer-0.66
Marks for Not Attempting0

Watch Video Discussion on Prelims 2024

See also: UPSC Prelims 2024 Question Paper Download

How to Download GS Paper – I Answer key?

On this web page, Rau’s IAS Team has prepared the GS Paper I answer key for you. This is an up-to-date page with the latest answer key respective to the Prelims paper of 2024, which was held on June 16th.

FAQs

What is the importance of the GS Paper I answer key?

The GS Paper I answer key holds immense importance for students who appeared for the UPSC Prelims exam on June 16th, 2024, as it serves as a crucial tool in their preparation for the UPSC Mains examination.

How reliable is the answer key provided by Rau’s IAS Team?

Rau’s IAS Team is a reputed coaching institute with experienced faculties and subject matter experts. Their answer keys are carefully prepared and cross-checked to ensure accuracy and reliability.

Can I use the answer key for self-evaluation?

Absolutely! The GS Paper I answer key is an excellent tool for self-evaluation. It allows you to identify the questions you answered correctly and the ones you struggled with, helping you pinpoint your areas of strength and weakness.

Is the answer key available for other papers as well?

Yes, Rau’s IAS Team provides answer keys for all the papers of the UPSC Civil Services Examination (GS & CSAT).

From this page, you can download a detailed explanation and answer key for the UPSC Prelims 2024. This will be helpful for checking your score and determining your accuracy.